You are on page 1of 111

Science Bowl Practice Questions Biology - 1

Science Bowl Practice Questions – Biology


1. Multiple Choice: Which of the following describes the major difference between bryophytes (pron:
bry-oh-fites) and tracheophytes (pron: tray-key-o-fites)?

a) tracheophytes can survive on land


b) tracheophytes have seeds inside fruits
c) tracheophytes can make their own food
d) tracheophytes have vessels to transport materials

ANSWER: D -- TRACHEOPHYTES HAVE VESSELS TO TRANSPORT


MATERIALS

2. Short Answer: Complete the sentence. The alternative forms of gene at the same locus on homologous
chromosomes are called what?

ANSWER: ALLELES

3. Short Answer: The human pelvic girdle is composed of 3 fused bones on each side for
a total of 6 bones. Name the 3 fused bones.

ANSWER: ILLIUM, ISCHIUM, and PUBIS

4. Short Answer: What are rings of muscle which CLOSE body openings such as the lips
called?

ANSWER: SPHINCTERS

5. Multiple Choice: The myofilaments of muscles consist primarily of two proteins. These two proteins
are called:

a) actin and myosin


b) progesterone and testosterone
c) progesterone and estrogen

ANSWER: A -- ACTIN AND MYOSIN

6. Short Answer: Is studies of the human body, what is used to describ a blood-filled sac formed by an
outpouching in an arterial or venous wall?

ANSWER: ANEURYSM
Science Bowl Practice Questions Biology - 2

7. Multiple Choice: What French scientist is considered the father of paleontology? Was it:

a) George Cuvier
b) Marquis de Condorcet
c) Jacque Tonnies
d) Auguste Compte

ANSWER: A -- GEORGE CUVIER

8. Multiple Choice: What Roman physician's studies of human anatomy based on lower animals served as
the only text on the subject for nearly 1 centuries? Was it:

a) Cartrum
b) Polonius
c) Galen
d) Monterius

ANSWER: C -- GALEN

9. Multiple Choice: What percentage of the atmosphere consists of carbon dioxide? Is it approximately:

a) .03% (read: three-hundredths of a percent)


b) 1%
c) 5%
d) 7%

ANSWER: A -- .03%

10. Short Answer: What process in all living things releases carbon dioxide as a waste product?

ANSWER: RESPIRATION

11. Short Answer: For convenience, living things are placed into variou groups. The taxonomic breakdown
of living things consists of the following categories: Family, Class, Genus, Phylum, Order, Kingdom
and Species. Give the order of these terms from least specific to most specific.

ANSWER: KINGDOM, PHYLUM, CLASS, ORDER, FAMILY, GENUS, SPECIES

12. Short Answer: In biology, what is the word used to describe the appearance of an individual without
regard to its hereditary constitution?

ANSWER: PHENOTYPE (pron: fee-no-type)


Science Bowl Practice Questions Biology - 3

13. Multiple Choice: The chromosomes responsible for characteristics other than sex are known by which
of the following terms?

a) ribosomes
b) lysosomes
c) autosomes
d) spermatocytes

ANSWER: C -- AUTOSOMES

14. Short Answer: What genetic term describes the situation when a part of a chromosome is broken off
and lost?

ANSWER: CHROMOSOMAL DELETION

15. Multiple Choice: Each of the following is a cell organelle except one. Which one of these is NOT a cell
organelle? Is it

a) mitochondrion
b) lysosome
c) cytoplasm
d) endoplasmic reticulum

ANSWER: C -- CYTOPLASM

16. Multiple Choice: When a color blind man marries a woman pure for normal color vision, it is probable
that one of the following situations may result. Is it probable that

a) all the children will be color blind


b) all the grandchildren will be color blind
c) only the sons will be colorblind
d) half the grandsons will be color blind

ANSWER: D -- HALF THE GRANDSONS WILL BE COLOR BLIND

17. Multiple Choice: Which of the following are skin receptors in humans that are sensitive to heat? Are
they:
a) end organs of Krause
b) Meissner's corpuscles
c) end organs of Ruffini
d) Pacinian corpuscles

ANSWER: C -- END ORGANS OF RUFFINI


Science Bowl Practice Questions Biology - 4

18. Multiple Choice: Into which of the following acids is glucose broken down in the first stage of
carbohydrate metabolism? Is it:

a) pyruvic acid (pie-rue-vick acid)


b) lactic acid
c) hydrochloric acid
d) citric acid

ANSWER: A -- PYRUVIC ACID

19. Multiple Choice: Hormones are composed from many classes of molecules. As far as our present
knowledge extends, hormones are NOT found in which of the following categories of substances:

a) proteins
b) peptides
c) nucleic acids
d) steroids

ANSWER: C -- NUCLEIC ACIDS

20. Short Answer: What is the term given to a fatty acid that has more than one set of double bonds
between carbons?

ANSWER: POLYUNSATURATED

21. Multiple Choice: The process by which an amino acid loses its amino group is called:

a) hydration
b) deamination
c) oxidoamination
d) dehydration

ANSWER: B – DEAMINATION

22. Multiple Choice: In a plasma membrane, which of the following provides a general barrier to the
movement of molecules?

a) lipids
b) proteins
c) carbohydrates
d) all of these

ANSWER: A – LIPIDS
Science Bowl Practice Questions Biology - 5

23. Multiple Choice: The name of the process by which oil glands in mammalian skins secrete oils is:

a) active transport
b) apocrine secretion
c) holocrine secretion
d) osmosis

ANSWER: C -- HOLOCRINE SECRETION

24. Multiple Choice: An individual with three X chromosomes is likely t be:

a) a clinically normal female


b) an abnormal female
c) a Turner's individual
d) a Kleinfelter's individual

ANSWER: B -- AN ABNORMAL FEMALE

25. Short Answer: Who is considered the father of Taxonomy?

ANSWER: (CAROLUS) LINNAEUS

26. Multiple Choice: In the first stage of photosynthesis, light energy is used to:

a) move water molecules


b) denature chlorophyll
c) split water
d) produce carbohydrates

ANSWER: C -- SPLIT WATER

27. Multiple Choice: One-celled algae enclosed in minute two-part silic shells are called:

a) dinoflagellates (pron: die-no-flaj-el-ates)


b) coelenterates (pron: sell-en-ter-ates)
c) annelids
d) diatoms

ANSWER: D – DIATOMS
Science Bowl Practice Questions Biology - 6

28. Multiple Choice: Osmoregulation is concerned with:

a) excretion
b) ionic regulation
c) control of the body's water content
d) carbon dioxide regulation

ANSWER: C -- CONTROL OF THE BODY'S WATER CONTENT

29. Multiple Choice: Which of the following is not an arachnid?

a) lobster
b) scorpion
c) black widow spider
d) tick

ANSWER: A -- LOBSTER

30. Multiple Choice: Which of the following is not found in blood?

a) fibrinogen
b) glucose
c) urea
d) glycogen

ANSWER: D -- GLYCOGEN

31. Multiple Choice: Proof that plants can grow and reproduce using onl inorganic nutrients comes from
the study of:

a) phototrophic nutrition
b) mycorrhiza
c) hydroponics
d) humus

ANSWER: C – HYDROPONICS

32. Multiple Choice: Water molecules enter plant epidermal cells by:

a) osmosis
b) active transport
c) translocation
d) transpiration

ANSWER: A – OSMOSIS
Science Bowl Practice Questions Biology - 7

33. Short Answer: What molecule is the energy source for virtually all of the work done by the cell?

ANSWER: ATP (Adenosine Triphosphate)

34. Multiple Choice: Which of the following diseases has been eradicate from the world?

a) cholera
b) smallpox
c) diphtheria
d) poliomyelitis

ANSWER: B -- SMALLPOX

35. Multiple Choice: We believe the beginnings of life on earth took place in the:

a) air
b) land masses
c) shallow oceans
d) glaciers

ANSWER: C -- SHALLOW OCEANS

36. Multiple Choice: Each antibody molecule is made up of how many PAIR of polypeptide chains, joined
together by disulfide bonds.

a) 1
b) 2
c) 3
d) 4

ANSWER: B – 2

37. Multiple Choice: The immune system normally discriminates between which types of antigens.

a) B and T cells
b) self and non-self
c) humoral and cell-mediated
d) primary and secondary

ANSWER: B -- SELF AND NON-SELF

38. Short Answer: Proteins that cause reactions to proceed more rapidly and lower the energy of activation
are called.

ANSWER: ENZYMES

39. Multiple Choice: What percent of the total radiant energy received by a plant is
converted to chemical energy?

a) less than 2
b) 10
c) 50
d) 75

ANSWER: A -- LESS THAN 2

40. Multiple Choice: Genes are able to control a cell's fate by determining the synthesis of:

a) other genes
Science Bowl Practice Questions Biology - 8

b) enzymes
c) carbohydrates
d) fat

ANSWER: B -- ENZYMES

41. Multiple Choice: Resolution by any microscope is ULTIMATELY limited by the:

a) curvature of the lenses


b) wavelength of the light
c) light intensity
d) lens aberrations

ANSWER: B -- WAVELENGTH OF THE LIGHT

42. Multiple Choice: Humans cannot digest cellulose because:

a) it does not contain sugars


b) it is made up of disaccharides
c) it is made up of monosaccharides
d) humans lack the proper enzymes

ANSWER: D -- HUMANS LACK THE PROPER ENZYMES

43. Multiple Choice: Cholesterol, testosterone, and estrogen are all examples of:

a) proteins
b) steroids
c) nucleic acids
d) alcohols

ANSWER: B -- STEROIDS

44. Short Answer: What is sometimes called animal starch?

ANSWER: GLYCOGEN

45. Short Answer: How many PAIRS of nerves leave the vertebrate brain?

ANSWER: 12

46. Short Answer; The vertebrate eyeball is bounded anteriorly by what convex, transparent object?

ANSWER: CORNEA

47. Short Answer: What are the names given to the basic subunits of a nucleic acid.

ANSWER: MONONUCLEOTIDES or NUCLEOTIDES

48. Short Answer: What is the term given to enzymes whose action is modulated by binding of a molecule
to a site other than the active site?

ANSWER: ALLOSTERIC ENZYMES

49. Multiple Choice: The muscle attachment to the bone of lesser movement is called the muscle's:

a) insertion
b) head
c) origin
d) tail
Science Bowl Practice Questions Biology - 9

ANSWER: C -- ORIGIN

50. Short Answer: DNA present in a structural gene that does not seem to have a specific coding function
is known as what?

ANSWER: INTRON

51. Multiple Choice: The organic portion of bone consists of which one of the following proteins.

a) fibrin
b) collagen
c) actin
d) myosin

ANSWER: B – COLLAGEN
Science Bowl Practice Questions Biology - 10

52. Multiple Choice: Octopus and squid belong to the class of molluscs known as:

a) cephalopoda (pron: sef-eh-lah-pod-ah)


b) bivalvia
c) gastropoda (pron: gas-tro-pod-ah)
d) polyplacophora (pron: poly-pla-so-for-ah)

ANSWER: A -- CEPHALOPODA

53. Short Answer: The name given to a structure present in angiosperms but lacking in gymnosperms is?

ANSWER: OVARY or OVULARY

54. Multiple Choice: In mammals, the primary function of the loop of Henle is:

a) reabsorption of water
b) water secretion
c) ammonia secretion
d) bicarbonate reabsorption

ANSWER: A -- REABSORPTION OF WATER

55. Short Answer: What is the name of the phenomenon that occurs when tissue culture is infected with
one type of virus that causes the culture to resist infection by a second type of virus?

ANSWER: VIRAL INTERFERENCE or INTERFERENCE

56. Short Answer: What are gas bubbles which form within plant cells o in blood known as?

ANSWER: EMBOLISM

57. Short Answer: What are the water conducting cells in non-flowering vascular plants know as?

ANSWER: TRACHEIDS

58. Short Answer: On a dry day, water diffuses out of the intercellula spaces of the leaf through what part
of the leaf?

ANSWER: STOMATA

59. Short Answer: A clogging of the bile duct interferes with the digestion of what category of food?

ANSWER: FATS or LIPIDS or FAT SOLUBLE VITAMINS

60. Short Answer: What is the name given to the jellylike substance filling the chamber behind the lens of
the human eye?

ANSWER: vitreous body vitreous humor

61. Short Answer: The Erythrocytes of mammals lack what?

ANSWER: nucleus or DNA or chromosomes or mitochondria

62. Multiple Choice: The name of the ductless glands which secrete thei product into the circulatory
system are:

a) exocrine (pron: ek-seh-kren)


b) apocrine (pron: ap-eh-kren)
c) holocrine (pron: hoe-leh-kren)
Science Bowl Practice Questions Biology - 11

d) endocrine (pron: en-deh-kren)

ANSWER: D -- ENDOcrine

63. Multiple Choice: In the first stage of photosynthesis, light energy splits water molecules leaving a by-
product which is released. This byproduct is:

a) hydrogen
b) carbon dioxide
c) oxygen
d) methane

ANSWER: C -- oxygen

64. Multiple Choice: The bacteria which cause dental cavities in humans break down sugars, releasing
what chemical, that causes tooth destruction?

a) acids
b) bases
c) enzymes
d) monosaccharides

ANSWER: A -- acids

65. Multiple Choice: Blood platelets are:

a) cytoplasmic fragments
b) free nuclei
c) a kind of red cell
d) found only during clot formation

ANSWER: A -- Cytoplasmic fragments


Science Bowl Practice Questions Biology - 12

66. Multiple Choice: Oxidation of fats and carbohydrates within a cell would be an example of:

a) anabolism
b) catabolism
c) biosynthesis
d) none of the above

ANSWER: B -- Catabolism

67. Multiple Choice: The muscle which is under involuntary control is:

a) striated
b) smooth
c) skeletal
ANSWER: B -- SMOOTH

68. Multiple Choice: The secondary pacemaker of the heart is known as the:

a) AV node
b) SA node
c) Purkinje Bundle
d) None of the above

ANSWER: A -- AV node

69. Multiple Choice: A pyrogen is a substance released during inflammation that aids in causing:

a) swelling
b) redness
c) fever
d) pain

ANSWER: C -- fever

70. Multiple Choice: Which of the following living creatures is most likely to have evolved from the
trilobite?

a) millipede
b) sea star
c) sea anemone
d) whelk

ANSWER: A – millipede
Science Bowl Practice Questions Biology - 13

71. Multiple Choice: A lake which has high nutrient availability is:

a) oligotrophic (pron: all-i-go-tro-fik)


b) eutrophic (pron: you-tro-fik)
c) mesotrophic (pron: mez-eh-tro-fik)

ANSWER: B -- eutropHic

72. Multiple Choice: The hormone causing growth of the endometrium (pron: en-doe-me-tree-um) is:

a) androgen (pron: ann-drah-jen)


b) leutenizing hormone (pron: lew-ten-I-zing)
c) estrogen
d) prolactin

ANSWER: C -- estrogen

73. Multiple Choice: Nitrogen fixation is accomplished by:

a) plants
b) bacteria
c) animals
d) viruses

ANSWER: B -- bacteria

74. Multiple Choice: The vesicles formed via the process known as phagocytosis (pron: fag-eh-seh-toe-sis)
are called:

a) lysosomes (pron: lye-sah-somes: somes rhymes with homes)


b) Golgi apparatus
c) food vacuoles
d) mitochondria (pron: my-toe-chon-dria)

ANSWER: C -- FOOD VACUOLES

75. Multiple Choice: Of the following three substances, which provides the LEAST source of energy for
the body?

a) protein
b) fats
c) carbohydrates

ANSWER: A – proteins
Science Bowl Practice Questions Biology - 14

76. Multiple Choice: Type A blood contains isohemaglutinins against which red cells?

a) type A
b) type B
c) type O
d) type G

ANSWER: A -- type A

77. Multiple Choice: Which cell count tends to be elevated when an individual has an allergy or parasitic
worms?

a) red blood cells


b) erythrocyte
c) eosinophil (pron: e-o-sin-o-fill)
d) platelet

ANSWER: C -- eosinophil

78. Multiple Choice: Veins tend to have the following except:

a) valves for unidirectional flow


b) very elastic walls
c) thin walls
d) increasing size toward the heart

ANSWER: B -- very elastic walls

79. Multiple Choice: Gallstones are most often the result of:

a) gallbladder irritation
b) eating cherries and swallowing the pits
c) precipitation of cholesterol
d) accumulated bile pigment

ANSWER: C -- precipitation of cholesterol

80. Multiple Choice: The loss of water by plants in the form of vapor i called:

a) evaporation
b) transpiration
c) evapotranspiration
d) vaporization

ANSWER: C – evapotranspiration
Science Bowl Practice Questions Biology - 15

81. Multiple Choice: Sponges are included in which of the following phyla?

a) Cnidaria (pron: nid-air-e-ah)


b) Porifera
c) Reptilia
d) Foraminifera (pron: feh-ram-eh-nif-eh-rah)

ANSWER: B -- PORIFERA

82. Short Answer: Name the tendon in humans which attaches the gastrocnemius and soleus muscles to the
calcaneum or heel bone.

ANSWER: Achilles Tendon

83. Short Answer: What is the normal pH of human blood?

ANSWER: 7.4

84. Short Answer: Myosin is one of two proteins that make up the myofibrils of striated muscles. Name the
other protein.

ANSWER: Actin

85. Short Answer: Part of the endocrine system in humans, these two glands are small bodies located at the
upper end of each kidney. While these glands perform a variety of functions, two of the most important
are (1) control of the body's adjustment to an upright posture, and (2) accommodation of the body to
intermittent rather than constant intake of food. Name these glands.

ANSWER: Adrenal Glands

86. Short Answer: Name the gelatin-like substance which is prepared for various species of red algae
growing in Asiatic waters. The prepared product appears in the form of cakes, coarse granules, long
shreds or in thin sheets. It is used extensively alone or in combination with various nutritive substances,
as a medium for culturing bacteria and various fungi

ANSWER: Agar-agar or just Agar

87. Short Answer: What is a partial or total loss of memory of a temporary or permanent nature known as?

ANSWER: Amnesia
Science Bowl Practice Questions Biology - 16

88. Multiple Choice: All of the following are characteristic of amphibians EXCEPT:

a) moist skin
b) the absence of scales
c) metamorphosis
d) live in salt water

ANSWER: D -- live in salt water

89. Short Answer: What term is used to describe the procedure whereby a catheter is inserted into an
individual's heart, a radio-opaque medium is injected, and x-ray images are made. The procedure is
used to locate where arteries are blocked and the degree to which the blockage has developed.

ANSWER: Angiography or Angiogram

90. Multiple Choice: Which of the following joints is formed by the articulation of the tibia, the malleolus
of the fibula, and the convex surface of the talus. Is it the:

a) ankle
b) knee
c) elbow
d) wrist

ANSWER: A -- Ankle

91. Multiple Choice: A plant which completes its life cycle, from seed to seed, in a single growing season
is known as:

a) perennial
b) cyclic
c) annual
d) circadian

ANSWER: C – Annual

92. Multiple Choice: In flowers the terminal part of a stamen, containing the pollen sacs is known as the:

a) anther
b) style
c) filament
d) pistil

ANSWER: A -- Anther

93. Multiple Choice: The process of nuclear reorganization in protozoa in which the nucleus divides, each
half undergoes a maturation, and the tw persisting functional nuclei reunite is known as:

a) autogamy
b) autolysis
c) autoregression
d) autotomy

ANSWER: A -- Autogamy

94. Multiple Choice: Of the following, which is NOT a bacterial disease Is it:

a) strep throat
b) herpes
c) Bubonic Plague
Science Bowl Practice Questions Biology - 17

d) Diphtheria

ANSWER: B -- herpes

95. Multiple Choice: Of the following, which is NOT a viral disease? Is it:

a) Hepatitis
b) Chicken Pox
c) Tuberculosis
d) Rabies

ANSWER: C – Tuberculosis

96. Multiple Choice: Most of the fungi commonly observed are members of this group of fungi, which
includes toadstools, mushrooms and puffballs. This group of fungi is known as:

a) Phycomycetes (pron: fI-kO-my-seats)


b) Ascomycetes (pron: as-kO-my-seats)
c) Basidomycetes (pron: beh-sid-E-O-my-seats)
d) Plyocetes (pron: ply-O-seats)

ANSWER: C -- Basidomycetes

97. Multiple Choice: Truffles, morels and the many yeasts which are basic to fermentation processes
belong to this group of fungi. These fung are known as:

a) Phycomycetes
b) Ascomycetes
c) Basidomycetes
d) Plyocetes

ANSWER: B -- Ascomycetes

98. Multiple Choice: Many living organisms exhibit the unique property of producing visible light. What
term is used to describe this phenomena?

ANSWER: Bioluminescence or photoluminescence

99. Multiple Choice: Many living organisms exhibit the unique property of producing visible light. The
compound that is oxidized with subsequent light emission is usually referred to as luciferin. What is the
enzyme which catalyzes the reaction known as?

ANSWER: Luciferase

100. Multiple Choice: The muscle which RAISES a bird's wing is known as the:

a) Pectoralis major
b) pectoralis minor
c) soleus
d) gluteus maximus

ANSWER: B -- pectoralis minor


Science Bowl Questions Biology - 1

Science Bowl Questions – Biology, Set 2

1. Multiple Choice: The adult human of average age and size has approximately how many quarts of
blood? Is it:

a) 4
b) 6
c) 8
d) 10

ANSWER: B -- 6

2. Multiple Choice: Once the erythrocytes enter the blood in humans, it is estimated that they have an
average lifetime of how many days. Is it:

a) 10 days
b) 120 days
c) 200 days
d) 360 days

ANSWER: B -- 120 Days

3. Multiple Choice: Of the following, which mechanisms are important in the death of erythrocytes (pron:
eh-rith-reh-sites) in human blood? Is it

a) phagocytosis (pron: fag-eh-seh-toe-sis)


b) hemolysis
c) mechanical damage
d) all of the above

ANSWER: D -- all of the above

4. Multiple Choice: Surplus red blood cells, needed to meet an emergency, are MAINLY stored in what
organ of the human body? Is it the:

a) pancreas
b) spleen
c) liver
d) kidneys

ANSWER: B – spleen

5. Multiple Choice: When a human donor gives a pint of blood, it usually requires how many weeks for
the body RESERVE of red corpuscles to be replaced? Is it:

a) 1 week
b) 3 weeks
c) 7 weeks
d) 21 weeks

ANSWER: C -- 7 weeks

6. Short Answer: There are three substances found in human blood which carry oxygen and which begin
with the letter "H". Name two of these substances.

ANSWER: Hemoglobin, Hemocyanin, Hemerythrin

7. Multiple Choice: The several types of white blood cells are sometime collectively referred to as:
Science Bowl Questions Biology - 2

a) erythrocytes (pron: eh-rith-row-cites)


b) leukocytes (pron: lew-kah-cites)
c) erythroblasts (pron: eh-rith-rah-blast)
d) thrombocytes (pron: throm-bow-cites)

ANSWER: B -- leukocytes

8. Multiple Choice: The condition in which there is a DECREASE in the number of white blood cells in
humans is known as:

a) leukocytosis (pron: lew-kO-sigh-toe-sis)


b) leukopenia (pron: lew-kO-pea-nee-ah)
c) leukemia (pron: lew-kee-me-ah)
d) leukohyperia (pron: lew-kO-high-per-e-ah)

ANSWER: B -- leukopenia

9. Multiple Choice: The smallest of the FORMED elements of the blood are the:

a) white cells
b) red cells
c) platelets
d) erythrocytes

ANSWER: C – platelets

10. Multiple Choice: Which of the following statements concerning platelets is INCORRECT. Platelets:

a) contain DNA
b) are roughly disk-shaped
c) have little ability to synthesize proteins
d) are between 1/2 and 1/3 the diameter of the red cell

ANSWER: A -- contain DNA

11. Short Answer: What is the primary function of the platelets in huma blood?

ANSWER: clotting or blocking leaks from blood vessels

12. Multiple Choice: When a wound occurs in humans, the platelets in the blood activate a substance
which starts the clotting process. The substance which starts the clotting is:

a) adenosine (pron: ah-den-ah-seen)


b) histamine
c) lecithin (pron: less-ah-thin)
d) thrombin

ANSWER: D -- Thrombin

13. Multiple Choice: When looking at the cross section of the human tibia, one finds the RED marrow in
the:

a) medullary cavity
b) cancellous bone
c) periosteum
d) epiphysis

ANSWER: A -- medullary cavity


Science Bowl Questions Biology - 3

14. Multiple Choice: Lengthening of long bones in humans occurs in a particular area of the bone. This
area is called the:

a) medullary canal
b) cancellous bone
c) periosteum (pron: per-E-ahs-tee-em)
d) epiphysis (pron: eh-pif-eh-sis)

ANSWER: D – Epiphysis

15. Multiple Choice: The part of the human brain which is an important relay station for the sensory
impulses and also is the origin of many of the involuntary acts of the eye such as the narrowing of the
pupil in bright light is the:

a) hypothalamus
b) midbrain
c) corpus callosum
d) cerebellum

ANSWER: B -- Midbrain

16. Multiple Choice: In the human brain, body temperature, metabolism, heart rate, sexual development,
sleep and the body's use of fat and water are influenced by this region of the brain. This region of the
brain is the:

a) hypothalamus
b) midbrain
c) corpus callosum
d) cerebellum

ANSWER: A -- hypothalamus

17. Multiple Choice: In which cerebral lobes is the speech center located? Is it the:

a) frontal
b) parietal
c) temporal
d) occipital

ANSWER: A -- frontal

18. Multiple Choice: In most axons, the myelin sheath is interrupted at intervals of about 1 millimeter or
more. These interruptions are called the:

a) glial
b) nodes of Ranvier (pron: ron-vee-ay)
c) collaterals
d) nodes of Babinet

ANSWER: B -- Nodes of Ranvier

19. Short Answer: Mosses and liverworts comprise this subdivision of plants. Name this plant subdivision.

ANSWER: Bryophytes (pron: bry-eh-fites) or Bryophyta


Science Bowl Questions Biology - 4

20. Short Answer: This disease, caused by infection with the gram-negative Yersinia pestis, is transmitted
by fleas from rats to humans What is the more common name for this disease?

ANSWER: Bubonic Plague or Black Death

21. Short Answer: In the mammalian body, this element plays many important roles. Try to identify this
element with the fewest number of clues. This element is required to insure the integrity and
permeability of cell membranes, to regulate nerve and muscle excitability, to help maintain normal
muscular contraction, and to assure cardiac rhythmicity. It also plays a essential role in several of the
enzymatic steps involved in blood coagulation and is the most important element of bone salt. Name
this element.

ANSWER: Calcium

22. Multiple Choice: What eight-letter name starting with the letter "O" is given to that branch of medical
science concerned with the study of tumors?

ANSWER: Oncology

23. Short Answer: In the more highly developed animals, such as humans this gas is used to regulate the
activity of the heart, the blood vessels, and the respiratory system. WORKING MUSCLES PRODUCE
A LARGE AMOUNT OF THI SUBSTANCE. Narcosis due to this gas is characterized by mental
disturbances which can include confusion, headache, low blood pressure and hypothermia. Name this
gas.

ANSWER: Carbon Dioxide or CO2

24. Multiple Choice: Cariology is the study of the:

a) human heart
b) tooth decay
c) kidneys
d) liver

ANSWER: B -- Tooth Decay

25. Short Answer: The larval form of butterflies and moths is more commonly known as what?

ANSWER: caterpillar

26. Short Answer: Name the sac-like, blind pouch of the large intestine, situated below the level of the
junction of the small intestine into the side of the large intestine. At the lower portion of this pouch one
finds the appendix.

ANSWER: Cecum or Caecum

27. Multiple Choice: During the final stage of cell division, the mitotic apparatus disappears, the
chromosomes become attenuated, the centrioles duplicate and split, the nuclear membrane becomes
reconstituted and the nucleolus reappears. This phase of cell division is known as:

a) prophase (pron: prO-phase)


b) metaphase
c) anaphase
d) telophase

ANSWER: D -- Telophase

28. Multiple Choice: In cell division, the phase following the metaphas is known as:
Science Bowl Questions Biology - 5

a) prophase
b) anaphase
c) telophase
d) extophase

ANSWER: B -- Anaphase

29. Short Answer: All cells of an organism find their lineage from a single fertilized cell. This single
fertilized cell is called what?

ANSWER: Zygote

30. Multiple Choice: Name the clear watery liquid that surrounds the brain and spinal cord and fills the
four cavities or ventricles of the brain.

ANSWER: Cerebrospinal Fluid

31. Multiple Choice: The order of insects which includes beetles is known as:

a) Coleoptera (pron: kO-lee-op-teh-rah)


b) Orthoptera (pron: or-tho-op-teh-rah)
c) Hymenoptera (pron: high-meh-nop-teh-rah)
d) Diptera (pron: dip-teh-rah)

ANSWER: A -- Coleoptera

32. Multiple Choice: This major protein component of connective tissue in mammals comprises most of
the organic matter of skin, tendons, bones, and teeth, and occurs as fibrous inclusions in most other
body structures. Is this material:

a) elastin
b) collagen
c) fatty acids
d) keratin

ANSWER: B -- collagen

33. Multiple Choice: Sickle cell anemia and Huntington's chorea are both:

a) virus-related diseases
b) bacteria-related diseases
c) congenital disorders
d) none of the above

ANSWER: C -- Congenital Disorders

34. Multiple Choice: In most species of Paramecium there are how many contractile vacuoles? Is it:

a) one
b) two
c) three
d) four

ANSWER: B – Two

35. Multiple Choice: The major fibrous proteins are:

a) peptone and edestin


b) glutelin and leucine
Science Bowl Questions Biology - 6

c) valine and lysine


d) myosin and actin

ANSWER: D -- Myosin and Actin

36. Short Answer: Name the outer portion of a stem or root, bounded externally by the epidermis, and
internally by the cells of the pericycle.

ANSWER: Cortex

37. Multiple Choice: Costal cartilage:

a) attach the ribs to the sternum


b) cover the ends of the femur
c) is found in the pinna of the ear
d) forms the intervertebral disks of the backbone

ANSWER: A -- Attach the ribs to the sternum

38. Multiple Choice: From which grandparent or grandparents did you inherit your mitochondria (pron:
my-toe-chon-dria)? Is it your:

a) mother's parents
b) paternal grandfather
c) grand mothers
d) maternal grandmother

ANSWER: D -- maternal grandmother

39. Multiple Choice: Which of the following are NOT part of a neuron?

a) synapse
b) axon
c) Nissl bodies
d) dendrite

ANSWER: A – SYNAPSE

40. Multiple Choice: The resting potential of a neuron is dependent on what two ions?

a) lead and calcium ions


b) calcium and phosphate ions
c) sodium and potassium ions
d) potassium and phosphate ions

ANSWER: C -- sodium and potassium ions

41. Multiple Choice: Which of the following is NOT a type of neuron?

a) sensory
b) motor
c) association
d) stimulatory

ANSWER: D -- STIMULATORY

42. Multiple Choice: Melatonin (pron: mel-eh-toe-nin) is produced by the:

a) skin
Science Bowl Questions Biology - 7

b) pineal gland
c) liver
d) pituitary gland

ANSWER: B -- PINEAL GLAND

43. Multiple Choice: Which of the following statements is TRUE of insulin? Is it:

a) secreted by the pancreas


b) a protein
c) involved in the metabolism of glucose
d) all of the above

ANSWER: D -- ALL OF THE ABOVE

44. Multiple Choice: Select the hormone INCORRECTLY paired with its target.

a) TSH - thyroid gland


b) ACTH - anterior pituitary
c) LH - ovary or testis
d) MSH - melanocytes (pron: meh-lan-o-cite)

ANSWER: D -- MSH – MELANOCYTES

45. Multiple Choice: Which of the following tissues secrete hormones?

a) pancreas
b) ovaries
c) gastro-intestinal tract
d) all of the above

ANSWER: D -- ALL OF THE ABOVE

46. Multiple Choice: Which of the following structures is directly attached to the ovary?

a) oviduct
b) uterus
c) suspensory ligaments
d) vagina

ANSWER: C -- SUSPENSORY LIGAMENTS

47. Multiple Choice: Fertilization of the ovum by the sperm usually occurs in the:

a) oviduct
b) vagina
c) uterus
d) ovary

ANSWER: A -- OVIDUCT
Science Bowl Questions Biology - 8

48. Multiple Choice: The corpeus luteum secretes:

a) HCG
b) LH
c) FSH
d) progesterone

ANSWER: D -- PROGESTERONE

49. Multiple Choice: Which of the following does sperm NOT travel through?

a) ureter
b) urethra
c) vas deferens
d) epididymus

ANSWER: A – URETER

50. Multiple Choice: The placenta in humans is derived from the:

a) embryo only
b) uterus only
c) endometrium and embryo
d) none of the above

ANSWER: C -- ENDOMETRIUM AND EMBRYO

51. Multiple Choice: The number of mature gametes resulting from meiosis in the female is:

a) 1
b) 2
c) 3
d) 4

ANSWER: A -- 1

52. Multiple Choice: Synapsis and crossing over of chromosomes occurs in which phases of meiosis?

a) Interphase
b) Prophase
c) Meterphase
d) Teleophase

ANSWER: B -- PROPHASE

53. Multiple Choice: A layer of dead skin cells is found in the:

a) subcutaneous tissue
b) dermis
c) epidermis
d) no dead cells are in the skin

ANSWER: C -- EPIDERMIS

54. Multiple Choice: Glial (pron: glee-el) cells are found in the:

a) muscular system
b) digestive system
c) endocrine system (pron: en-de-kren)
Science Bowl Questions Biology - 9

d) nervous system

ANSWER: D -- NERVOUS SYSTEM

55. Multiple Choice: Myelin sheaths are found:

a) surrounding tendons
b) covering the brain
c) covering muscles
d) around axons of neurons

ANSWER: D -- AROUND AXONS OF NEURONS

56. Multiple Choice: Which of the following is an INCORRECT statement about the parasympathetic
system?

a) It increases digestive action.


b) It is the fight or flight system.
c) slows breathing rate
d) establishes resting state

ANSWER: B -- IT IS THE FIGHT OR FLIGHT SYSTEM.

57. Multiple Choice: Which of the following is NOT a component of the human axial skeleton?

a) sternum
b) vertebral column
c) tarsals
d) skull

ANSWER: C -- TARSALS

58. Multiple Choice: Phalanges are found in the:

a) feet
b) skull
c) hip
d) chest

ANSWER: A – FEET

59. Multiple Choice: The phase of contraction of a muscle occurs when:

a) tropomyosin binds and releases actin


b) myosin binds and releases actin
c) actin binds and releases myosin
d) none of the above

ANSWER: B -- MYOSIN BINDS AND RELEASES ACTIN

60. Multiple Choice: Select the INCORRECT statement concerning the muscular system.

a) Bones contact other bones at joints.


b) Flexors decrease the angle of a joint.
c) Adductors move a limb away from the midline.
d) Tendons attach muscle to bone.

ANSWER: C -- ADDUCTORS MOVE A LIMB AWAY FROM THE MIDLINE.


Science Bowl Questions Biology - 10

61. Multiple Choice: Which type of muscle is a syncytium (pron: sin-sish-E-um)?

a) skeletal
b) cardiac
c) smooth
d) all of the above

ANSWER: A -- SKELETAL

62. Multiple Choice: When the potential difference across a membrane of a neuron equals the threshold,
what results?

a) movement of the membrane


b) action potential
c) relaxation
d) contraction

ANSWER: B -- ACTION POTENTIAL

63. Multiple Choice: What ions determine the resting potential of a nerve?

a) sodium and calcium


b) calcium and copper
c) potassium and calcium
d) sodium and potassium

ANSWER: D -- SODIUM AND POTASSIUM

64. Multiple Choice: Which structure does NOT play a part in the motion of cells?

a) microvilli
b) cilia
c) flagella
d) pseudopodia

ANSWER: A -- MICROVILLI

65. Multiple Choice: Bacteriophage (pron: back-teer-e-o-faj) are:

a) bacteria
b) bacteria precursors
c) viruses
d) agents which cause the production of bacteria

ANSWER: C -- VIRUSES

66. Multiple Choice: Which of the following is NOT a mode of genetic exchange within a bacterial
population?

a) conjugation
b) transduction
c) transformation
d) translation

ANSWER: D -- TRANSLATION

67. Multiple Choice: The blastula develops into the:

a) gastrula
Science Bowl Questions Biology - 11

b) morula
c) endoderm
d) zygote

ANSWER: A -- GASTRULA

68. Multiple Choice: Tissue differentiation begins at which stage?

a) zygote
b) morula
c) blastula
d) gastrula

ANSWER: D – GASTRULA

69. Multiple Choice: The nervous system develops from which germ layer?

a) ectoderm
b) mesoderm
c) endoderm
d) none of the above

ANSWER: A -- ECTODERM

70. Multiple Choice: During inspiration, the diaphragm moves:

a) down by contraction
b) down by relaxation
c) up by contraction
d) up by relaxation

ANSWER: A -- DOWN BY CONTRACTION

71. Multiple Choice: The valve between the right ventricle and the pulmonary artery is the:

a) mitral valve
b) semilunar valve
c) bicuspid valve
d) tricuspid valve

ANSWER: A -- SEMILUNAR VALVE

72. Multiple Choice: Which of the following is NOT a function of the kidney?

a) excretion of urea
b) regulation of fluids and electrolytes
c) elimination of toxic substances
d) defecation

ANSWER: D -- DEFECATION

73. Multiple Choice: When CO2 (carbon dioxide) is dissolved in water, it yields a solution that:

a) has acidic properties


b) has basic properties
c) is neutral

ANSWER: A -- HAS ACIDIC PROPERTIES


Science Bowl Questions Biology - 12

74. Multiple Choice: Digestion of carbohydrates begins where?

a) small intestines
b) colon
c) mouth
d) stomach

ANSWER: C -- MOUTH

75. Multiple Choice: Digestion of PROTEINS begins in which of the following human organs?

a) small intestines
b) colon
c) mouth
d) stomach

ANSWER: D -- STOMACH

76. Multiple Choice: Bile has what function in digestion?

a) emulsify lipids
b) digest proteins
c) gluconeogenesis (pron: glue-ko-nee-o-gen-e-sis)
d) digest carbohydrates

ANSWER: A -- EMULSIFY LIPIDS

77. Multiple Choice: Of the following, which is a basic need of all living things?

a) oxygen gas
b) light
c) hydrogen gas
d) water

ANSWER: D -- WATER

78. Multiple Choice: A botanist is most likely to study:

a) Monerans
b) Protistans (pron: pro-tis-tans)
c) Fungi
d) Virions

ANSWER: C -- FUNGI

79. Multiple Choice: A virus must do what to reproduce?

a) form a latent virus


b) undergo transformation
c) infect a cell
d) conjugate

ANSWER: C -- INFECT A CELL

80. Multiple Choice: The chromosomes of a eukaryotic cell are located i the:

a) mitochondria (pron: my-toe-kon-dria)


b) nucleus
c) ribosome
Science Bowl Questions Biology - 13

d) endoplasma

ANSWER: B -- NUCLEUS

81. Multiple Choice: Which of the following is an example of symbiosis?

a) lichen
b) slime mold
c) amoeba
d) moss

ANSWER: A -- LICHEN

82. Multiple Choice: Oxygen enters the body of a grasshopper through:

a) gills
b) spinnerets
c) spiracles
d) book lungs

ANSWER: C -- SPIRACLES

83. Multiple Choice: A heart with a single atrium and single ventricle is a characteristic of adult:

a) amphibians
b) arthropods
c) birds
d) fish

ANSWER: D -- FISH

84. Short Answer: Name the four main excretory organs identified in man.

ANSWER: SKIN, KIDNEYS, LUNGS, INTESTINAL TRACT

85. Multiple Choice: The physical appearance and properties of an organism which is the expression of the
genetic makeup is called the:

a) phenotype
b) pangenesis
c) parental trait
d) genotype

ANSWER: A -- PHENOTYPE

86. Short Answer: How many nucleotides make up a codon (pron: kO-don)?

ANSWER: 3

87. Multiple Choice: The complex of sugar polymers and proteins which are patchily distributed on the
plasma membranes of animal cells is called

a) cellulose
b) chitin
c) glyocalyx
d) cytoskeleton

ANSWER: C -- GLYOCALYX
Science Bowl Questions Biology - 14

88. Short Answer: During cellular respiration, glucose is oxidized completely to what two compounds?

ANSWER: CO2 (CARBON DIOXIDE) AND H2O (WATER)

89. Multiple Choice: Organisms with cells containing two sets of parental chromosomes are called:

a) diploid
b) bisomal
c) haploid
d) autosomal

ANSWER: A – DIPLOID

90. Multiple Choice: The type of gene interaction in which the effects of one gene override or mask the
effects of other entirely different genes is called:

a) linkage
b) mutation
c) pleitropy (pron: ply-ah-tropy)
d) epistasis (pron: eh-pis-te-sis)

ANSWER: D – EPISTASIS

91. Multiple Choice: For which of the following creatures is fat the greatest percentage of body weight?

a) termite
b) blue whale
c) zebra
d) female lion

ANSWER: B -- BLUE WHALE

92. Multiple Choice: Which is false regarding freshwater fish?

a) their blood is hypertonic to their environment


b) they often actively take up salt
c) they excrete urine hypotonic to the blood
d) their gills actively excrete salts

ANSWER: D -- THEIR GILLS ACTIVELY EXCRETE SALTS

93. Multiple Choice: Neutral fats, oils and waxes may be classified as:

a) lipids
b) carbohydrates
c) proteins
d) none of the above

ANSWER: A -- LIPIDS

94. Short Answer: Name three basic morphologies of bacteria.

ANSWER: (1) COCCI (COCCUS), (2) BACILLI (BACILLUS) or ROD, (3) SPIROCHETES or
SPIRILLA or SPIRAL

95. Short Answer: What is the name of the 6 carbon monosaccharide that is the universal
cellular fuel of plants and animals?

ANSWER: GLUCOSE (DEXTROSE)


Science Bowl Questions Biology - 15

96. Multiple Choice: During which phase of the cell cycle are normal components of the
cell synthesized and assembled?

a) the M phase
b) the G1 phase
c) the S phase
d) the G2 phase

ANSWER: B -- THE G1 PHASE

97. Multiple Choice: Which of the following is NOT a characteristic shared by most of the
members of the kingdom plantae?

a) they are multicellular


b) they are nonmotile
c) they possess bilateral symmetry
d) there is an alternation of haploid and diploid generations

ANSWER: C -- THEY POSSESS BILATERAL SYMMETRY

98. Short Answer: If an individual has two dissimilar alleles for a trait, with regard to that trait he is said to
be:

ANSWER: HETEROZYGOUS

99. Short Answer: How many chromosomes per cell does a Down's Syndrome (trisomy 21) victim have?

ANSWER: 47

100. Short Answer: If a male who is heterozygous for an autosomal trait mates with a female who is
also heterozygous for that trait, what percent of their offspring are likely to be heterozygous for this
trait as well?

ANSWER: 50%
Science Bowl Practice Questions Chemistry - 1

Science Bowl Practice Questions - Chemistry

1. Multiple Choice: Which of the following scientists was awarded the Nobel Prize in 1911 for the
discovery of the radioactive elements, radium and polonium?

w) John Dalton
x) Dmitri Mendeleev
y) Emil Fischer
z) Marie Curie

ANSWER: Z -- MARIE CURIE

2. Short Answer: Who was awarded a Nobel Prize in 1911 for the discovery of the radioactive elements,
radium and polonium?

ANSWER: D -- MARIE CURIE

3. Multiple Choice: Which of the following pairs of German scientists discovered the atomic fission of
uranium and set the stage for the nuclear age?

w) Hahn and Strassman


x) Nernst and Planck
y) Ostwald and Heisenberg
z) Bohr and Rutherford

ANSWER: W -- HAHN AND STRASSMAN

4. Multiple Choice: Name the person who developed a table of elements which revealed regularities in
elemental properties in 1869?

w) Theodore Richards
x) Antoine Lavoisier
y) Dmitri Mendeleev
z) Svante Arrhenius

ANSWER: Y -- DMITRI MENDELEEV

5. Short Answer: Name the person who developed a table of elements which revealed regularities in
elemental properties?

ANSWER: DMITRI MENDELEEV


Science Bowl Practice Questions Chemistry - 2

6. Multiple Choice: Who was the first American chemist to receive a Nobel Prize? He was selected in
1914 for his precise determination of atomic weights.

w) Edward Frankland
x) Theodore Richards
y) John Bardeen
z) Paul Dirac

ANSWER: X -- THEODORE RICHARDS

7. Multiple Choice: How many electrons can occupy an s orbital?

w) one
x) two
y) three
z) four

ANSWER: X -- TWO

8. Short Answer: What is the maximum number of electrons that can occupy an s orbital?

ANSWER: TWO

9. Multiple Choice: Which of the following is a necessary characteristic for electrons that share an s
orbital?

w) coplanar spins
x) parallel spins
y) opposite spins
z) none of these

ANSWER: Y -- OPPOSITE SPINS

10. Multiple Choice: Which of the following terms refers to the number of molecules or ions attached to a
central metallic atom?

w) coordination number
x) connection number
y) bonding number
z) none of these

ANSWER: W -- COORDINATION NUMBER


Science Bowl Practice Questions Chemistry - 3

11. Multiple Choice: Which of the following pairs do NOT show similar chemical properties?

w) fluorine-argon
x) beryllium-aluminum
y) boron-silicon
z) lithium-magnesium

ANSWER: W -- FLUORINE-ARGON

12. Short Answer: What is the common name for calcium oxide, CaO?

ANSWER: QUICKLIME (LIME)

13. Multiple Choice: Which of the following terms refers to a material which slows neutrons?

w) retarder
x) moderator
y) buffer
z) bottleneck

ANSWER: X -- MODERATOR

14. Multiple Choice: Which of the following refers to the scattering of light by colloidal particles?

w) Rutherford effect
x) Tyndall effect (pron: tin-dahl)
y) Thompson effect
z) none of these

ANSWER: X -- TYNDALL EFFECT

15. Short Answer: Who is accredited in the history of modern chemistry of proving that neutrons existed?

ANSWER: JAMES CHADWICK

16. Multiple Choice: Which of the following refers to an easily melted product of the reaction between the
flux and the impurities of an ore?

w) slag
x) slurry
y) slug
z) none of the above

ANSWER: W – SLAG
Science Bowl Practice Questions Chemistry - 4

17. Short Answer: Name the thermodynamic state function that is a measure of randomness.

ANSWER: ENTROPY

18. Short Answer: What term is used to describe silver which contains 7.5% copper?

ANSWER: STERLING

19. Short Answer: Name the state function that represents the heat content of a system.

ANSWER: ENTHALPY

20. Multiple Choice: Which of the following is NOT a physical property of crystalline boron?

w) brown color
x) diamond hardness
y) brittleness
z) high electrical resistance

ANSWER: W -- BROWN COLOR (Crystalline boron is transparent.)

21. Multiple Choice: Which noble gas has the highest melting point ?

w) argon
x) krypton
y) xenon
z) radon

ANSWER: Z – RADON

22. Multiple Choice: The noble gases have low boiling points because:

w) their ionization potentials are very low


x) only weak Van der Waals forces hold the atoms together
y) they are produced by the fractional distillation of liquid air
z) they have eight electrons in the outer shell

ANSWER: X -- ONLY WEAK VAN DER WAALS FORCES HOLD THE ATOMS
TOGETHER
Science Bowl Practice Questions Chemistry - 5

23. Multiple Choice: For which of the following classes of salts are all of its compounds soluble in water?

w) sulfates
x) nitrates
y) hydroxides
z) chlorides

ANSWER: X -- NITRATES

24. Multiple Choice: Which of the following is NOT a general property of hydroxide bases in aqueous
solution?

w) High solubility
x) Bitter taste
y) Ability to neutralize aqueous acid solutions
z) Ability to change phenolphthalein from colorless to red

ANSWER: W -- HIGH SOLUBILITY (The hydroxides of the alkali metals are soluble in water but all the
other metal hydroxides are only moderately or sparingly soluble.)

25. Multiple Choice: Aqua regia is:

w) concentrated nitric acid


x) a mixture of three parts of concentrated hydrochloric acid and one part concentrated
nitric acid
y) concentrated hydrochloric acid
z) a mixture of sulfuric acid and hydrochloric acid

ANSWER: X -- A MIXTURE OF THREE PARTS OF CONCENTRATED HYDROCHLORIC ACID


AND ONE PART CONCENTRATED NITRIC ACID

26. Short Answer: What is the common name given to a mixture of three parts of concentrated
hydrochloric acid and one part concentrated nitric acid?

ANSWER: AQUA REGIA

27. Multiple Choice: A solution with a pH of 12 is:

w) weakly acidic
x) strongly acidic
y) weakly basic
z) strongly basic

ANSWER: Z -- STRONGLY BASIC


Science Bowl Practice Questions Chemistry - 6

28. Multiple Choice: The halogen with the largest atomic radius is:

w) Cl
x) I
y) Br
z) At

ANSWER: Z -- At

29. Short Answer: Which halogen has the largest atomic radius:

ANSWER: ASTATINE

30. Multiple Choice: Compounds of iodine are NOT used for which of the following?

w) as a refrigerant
x) in photographic film
y) as an essential part of diet
z) as an antiseptic

ANSWER: W -- AS A REFRIGERANT

31. Multiple Choice: Allotropic modifications of phosphorus include each of the following EXCEPT:

w) red phosphorus
x) black phosphorus
y) white phosphorus
z) green phosphorus

ANSWER: Z -- GREEN PHOSPHORUS

32. Short Answer: How many milligrams are in one metric carat?

ANSWER: 200 MILLIGRAMS

33. Short Answer: What class of organic compounds is characterized by an -OH group bonded to an alkyl
group?

ANSWER: ALCOHOL
Science Bowl Practice Questions Chemistry - 7

34. Multiple Choice: Which of the following contain sp2 hybridized carbon atoms?

w) alkanes
x) alkenes
y) alkynes
z) cycloalkanes

ANSWER: X -- ALKENES

35. Multiple Choice: Which of the following phrases is NOT appropriate for a description of benzene?

w) characteristic reactivity of an alkene


x) volatile liquid
y) aromatic hydrocarbon
z) resonance hybridization

ANSWER: W -- CHARACTERISTIC REACTIVITY OF AN ALKENE

36. Multiple Choice: Which of the following is NOT an aromatic hydrocarbon?

w) naphthalene
x) hexane
y) styrene
z) xylene

ANSWER: X -- HEXANE

37. Multiple Choice: Which of the following is NOT an aliphatic hydrocarbon?

w) n-octane
x) propylene
y) n-tridecane
z) benzene

ANSWER: Z -- BENZENE

38. Multiple Choice: Which of the following is NOT a means of preparing carbon dioxide?

w) burning coke in the absence of air


x) combustion of methane
y) heating certain normal carbonates
z) action of acids on carbonates

ANSWER: W -- BURNING COKE IN THE ABSENCE OF AIR


Science Bowl Practice Questions Chemistry - 8

39. Short Answer: After carbon, hydrogen, and oxygen, the next most abundant element in proteins is:

ANSWER: NITROGEN

40. Multiple Choice: One of the following is NOT a lipid. Is it:

w) fats and oils


x) starches
y) steroids
z) waxes

ANSWER: X – STARCHES

41. Multiple Choice: Which one of the following is NOT a vitamin?

w) ascorbic acid
x) niacin (pron: nI-eh-sen)
y) trypsin (pron: trip-sen)
z) riboflavin (pron: rye-bow-fla-vin)

ANSWER: Y -- TRYPSIN

42. Multiple Choice: Of the following, which has units of "Pascal".

w) force
x) volume
y) pressure
z) viscosity

ANSWER: Y -- PRESSURE

43. Multiple Choice: A Poise is the unit of measure of which of the following?

w) pressure
x) viscosity
y) force
z) mass

ANSWER: X – VISCOSITY
Science Bowl Practice Questions Chemistry - 9

44. Multiple Choice: Which of the following is the element represented by the symbol B?

w)Barium
x)Bismuth
y)Boron
z)Beryllium

ANSWER: Y -- BORON

45. Short Answer: Which element is represented by the symbol B?

ANSWER: BORON

46. Short Answer: What is the element represented by the symbol Rb?

ANSWER: RUBIDIUM

47. Short Answer: What is the element represented by the symbol Sn?

ANSWER: TIN

48. Short Answer: What is the element represented by the symbol Sr?

ANSWER: STRONTIUM

49. Multiple Choice: In the periodic table, which of the following identifies a horizontal row?

w) period
x) group
y) family
z) series

ANSWER: W -- PERIOD

50. Multiple Choice: Which of the following describes the orientation of bonds in an sp3 hybridized atom?

w)triagonal
x)linear
y)tetrahedral
z)planar

ANSWER: Y – TETRAHEDRAL
Science Bowl Practice Questions Chemistry - 10

51. Multiple Choice: Forces in ionic crystalline solids are primarily which of the following?

w)covalent
x)electrostatic
y)Van der Waals
z)magnetic

ANSWER: X – ELECTROSTATIC

52. Multiple Choice: In which of the following is the "octet rule" violated?

w) CO2
+
x) NH4 (read: N - H - 4 - plus)
2-
y) SO4 (read: S - O - 4 - 2 minus)
z) NO2

ANSWER: Z -- NO2

53. Multiple Choice: Normality is defined as the number of:

w) moles of solute per 1000 grams of solvent


x) equivalent weights of solute per liter of solution
y) kilograms of solute per liter of solution
z) moles of solute per 1000 grams of solution

ANSWER: X -- EQUIVALENT WEIGHTS OF SOLUTE PER LITER OF SOLUTION

54. Multiple Choice: The oxidation number of N in NaNO2 is:

w) +5
x) +2
y) +3
z) -3

ANSWER: Y -- +3 (Accept 3)

55. Short Answer: What is the oxidation number of nitrogen in NaNO2:

ANSWER: +3 (Accept 3)
Science Bowl Practice Questions Chemistry - 11

56. Multiple Choice: Which one of the following atoms has two unpaired electrons in its ground state:

w) Sodium
x) Calcium
y) Oxygen
z) Aluminum

ANSWER: Y – OXYGEN

57. Multiple Choice: In which of the following compounds is the bonding most covalent?

w) NaCl
x) F2
y) MgS
z) Fe3C

ANSWER: X -- F2

58. Multiple Choice: The elements in group 1A of the periodic table are called:

w) halogens
x) alkaline earths
y) alkali metals
z) rare earths

ANSWER: Y -- ALKALI METALS

59. Short Answer: What is the family name given to the elements in group 1A of the periodic table?

ANSWER: ALKALI METALS

60. Short Answer: What is the name of the organic compound whose chemical name is amygdalin (pron: a-
MIG-da-lin) and is found in the pits of apricots and peaches?

ANSWER: LAETRILE

61. Multiple Choice: Name the element discovered upon its extraction from pitchblende?

w) platinum
x) uranium
y) iridium
z) plutonium

ANSWER: X -- URANIUM

62. Short Answer: All proteins contain at least 5 specific elements. Four of them are carbon, oxygen,
hydrogen, and nitrogen. What is the fifth element?

ANSWER: SULFUR

63. Multiple Choice: Cinnabar, pyrite and galena are minerals which consist of one or more metals
combined with the same element. Which one of the following is the element that is common to all three
of these minerals? Is it:

w) iron
x) copper
y) sulfur
z) sodium
Science Bowl Practice Questions Chemistry - 12

ANSWER: C-- SULFUR

64. Multiple- choice: In a flame test, the presence of copper in a solution is evident by what color flame? Is
the flame

w) red
x) orange
y) indigo
z) blue-green

ANSWER: Z -- BLUE-GREEN

65. Multiple Choice: Alpha rays are different from beta rays in one of the following ways. Are they
different since:

w) alpha rays are deflected in a magnetic field and beta rays are not
x) alpha rays are identical to the electron in mass, and beta rays are not
y) alpha rays have a positive charge and beta rays have a negative charge
z) alpha rays are produced within the nucleus and beta rays are produced outside the nucleus

ANSWER: Y -- ALPHA RAYS HAVE A POSITIVE CHARGE AND BETA RAYS HAVE A
NEGATIVE CHARGE

66. Multiple Choice: Which one of the following statements is TRUE regarding the molar heat capacity for
the noble gases?

w) heat capacity decreases from Helium to Xenon


x) heat capacity increases from helium to argon
y) heat capacity increases from Helium to Argon, then decreases to Xenon
z) heat capacity is the same for all noble gases

ANSWER: Z -- HEAT CAPACITY IS THE SAME FOR ALL NOBLE GASES


Science Bowl Practice Questions Chemistry - 13

67. Multiple Choice: Of the following types of material, which is the one that is heterogeneous? Is it:

w) quartz
x) feldspar
y) granite
z) mica

ANSWER: Y -- GRANITE

68. Multiple Choice: Which of the following halogens does NOT replace the other halogens from their
compounds?

w) fluorine
x) iodine
y) bromine
z) chlorine

ANSWER: X – IODINE

69. Short Answer: Excluding Astatine, which halogen does NOT replace the other halogens from their
compounds?

ANSWER: IODINE

70. Multiple Choice: The bond in hydrogen chloride is which of the following types? Is it

w) ionic
x) nonpolar covalent
y) polar covalent
z) covalent network

ANSWER: Y -- POLAR COVALENT

71. Multiple Choice: The bond in Fluorine is which of the following types? Is it

w) ionic
x) nonpolar covalent
y) polar covalent
z) covalent network

ANSWER: X -- NONPOLAR COVALENT


Science Bowl Practice Questions Chemistry - 14

72. Multiple Choice: Which of the following concentration expressions is defined as the number of moles
of solute per kilogram of solvent:

w) molarity
x) molality
y) normality
z) equivalents

ANSWER: X – MOLALITY

73. Multiple Choice: Among the following, the substance that is considered to be the earliest stage in coal
formation is:

w) sub-bituminous
x) bituminous
y) anthracite
z) lignite

ANSWER: Z – LIGNITE

74. Multiple Choice: When an aldehyde is heated with Fehling's solution, the aldehyde undergoes one of
the following processes. Does it undergo:

w) neutralization
x) oxidation
y) reduction
z) esterification

ANSWER: X -- OXIDATION

75. Multiple Choice: The hydrolysis of a fat using a solution of a strong hydroxide is called:

w) neutralization
x) esterification
y) saponification
z) condensation

ANSWER: Y -- SAPONIFICATION

76. Multiple Choice: In the common dry cell, the zinc atoms are:

w) reduced at the cathode


x) oxidized at the cathode
y) oxidized at the anode
z) reduced at the anode

ANSWER: Y -- OXIDIZED AT THE ANODE

77. Multiple Choice: As one goes across a period in the periodic table, the ionization energy generally
increases for one of the following reasons. Is it because of the increasing

w) atomic radius
x) melting point
y) nuclear charge
z) density

ANSWER: Y -- NUCLEAR CHARGE

78. Multiple Choice: Which of the following reactions involve NEITHER oxidation nor reduction?
Science Bowl Practice Questions Chemistry - 15

w) burning antimony in chlorine


x) decomposition of potassium chlorate
y) reaction of sodium with water
z) reaction of potassium hydroxide with nitric acid

ANSWER: Z -- REACTION OF POTASSIUM HYDROXIDE WITH NITRIC ACID

79. Short Answer: In a flame test both lithium and strontium produce various shades of this color. What is
this color?

ANSWER: RED

80. Multiple Choice: Who hypothesized that it would be impossible to know the exact location and exact
momentum of a particle at the same time? Was it:

w) Heisenberg
x) De Broglie
y) Planck
z) Schrodinger

ANSWER: A -- HEISENBERG

81. Short Answer: Who hypothesized that it would be impossible to know the exact location and exact
momentum of a particle at the same time?

ANSWER: HEISENBERG

82. Multiple Choice: Ammonia can be considered a Lewis base because it can donate:

w) electrons
x) protons
y) hydronium ions
z) hydroxide ions

ANSWER: A – ELECTRONS

83. Multiple Choice: Acids react with bases to form which of the following? Do they form:

w) salt and acidic anhydride


x) water and basic anhydride
y) basic anhydride and salt
z) salt and water

ANSWER: D -- SALT AND WATER

84. Multiple Choice: Which of the following is a metallic salt of a fatty acid? Is it:

w) soap
x) plastic
y) rubber
z) petroleum

ANSWER: A -- SOAP

85. Multiple Choice: Which of the following choices are considered to be polymers of amino acids? Are
they:

w) nucleotides
x) carbohydrates
Science Bowl Practice Questions Chemistry - 16

y) lipids
z) proteins

ANSWER: Z – PROTEINS

86. Multiple Choice: Which of the following pairs of compounds could NOT be used to create a buffer
solution? Would it be:

w) HCL and NaCl (read: H - C - L and N - A - C - L)


x) HCN and NaCN (read: H - C - N and N - A - C - N)
y) NH3 and NH4Cl (read: N - H - 3 and N - H - 4 - C - L)
z) HNO2 and NaNO2 (read: H - N - O -2 and N - A - N - O - 2)

ANSWER: W -- HCL AND NaCl (read: H - C - L and N - A - C - L)

87. Short Answer: In a body-centered structure for cesium chloride how many nearest neighbor chloride
ions surround a cesium ion?

ANSWER: 8

88. Short Answer: Metallic elements are often obtained from ores. Name the metallic element which is
produced from the ore Bauxite.

ANSWER: ALUMINUM
Science Bowl Practice Questions Chemistry - 17

89. Short Answer: Name any five of the six noble gases.

ANSWER: HELIUM, NEON, ARGON, KRYPTON, XENON, OR RADON

90. Short Answer: Name any four alkali metals.

ANSWER: LITHIUM, SODIUM, POTASSIUM, RUBIDIUM, CESIUM, AND FRANCIUM

91. Short Answer: The element I am thinking about has several interesting characteristics. It occupies the
same column in the periodic table as lead. Resistors which are commonly found in electronic circuits
are often made out of this element. In addition, in one of its forms, this element is one of the hardest
substances on earth. What element am I thinking about?

ANSWER: CARBON

92. Short Answer: Give me the names, in order of increasing mass, of the four lightest members the
halogen family.

ANSWER: FLUORINE, CHLORINE, BROMINE, AND IODINE

93. Short Answer: Ketones are organic compounds. Name the functional group that all ketone molecules
contain.

ANSWER: CARBONYL GROUP pr C DOUBLE BOND O

94. Short Answer: Four of the lightest ten elements in the periodic table are solids at standard temperature
and pressure. Name these four elements.

ANSWER: LITHIUM, BERYLLIUM, BORON, CARBON

95. Multiple Choice: Which of the following scientists first stated the law of conservation of mass? Was it:

w) Lavoisier
x) Dalton
y) Proust
z) Boyle

ANSWER: W -- LAVOISIER

96. Multiple Choice: Which of the following scientists explained the photoelectric effect? Was it:

w) Einstein
x) Planck
y) Bohr
z) Chadwick

ANSWER: W -- EINSTEIN

97. Multiple Choice: Which of the following scientists hypothesized that it would be possible for particles
to have wave-like properties ? Was it:

w) Planck
x) Heisenberg
y) Einstein
z) de Broglie

ANSWER: Z -- DE BROGLIE
Science Bowl Practice Questions Chemistry - 18

98. Short Answer: Who developed the equation that is used to determine the PROBABILITY of finding
the electron in any given place?

ANSWER: SCHRODINGER

99. Short Answer: How many hydrogen atoms are there in the organic compound 1,1,2,2-tetrabromoethane
(read: tetra - bromo - ethane)?

ANSWER: 2

100. Short Answer: Give the empirical formula for the organic compound naphthalene?

ANSWER: C10H8
Science Bowl Questions Chemistry - 1

Science Bowl Questions – Chemistry, Set 2


1. Multiple Choice: The triple bond between the carbon atoms causes acetylene, C2H2, to have which of
the following shapes?

w) trigonal planar (pron: try-gon-al)


x) linear
y) tetrahedral
z) trigonal bipyramidal

ANSWER: X -- LINEAR

2. Short Answer: What is the EMPIRICAL formula for glucose?

ANSWER: CH2O (read: C - H - 2 - O)

3. Multiple Choice: Which of the following four elements has the largest atomic radius? Is it:

w) strontium
x) francium
y) calcium
z) bromine

ANSWER: X -- FRANCIUM

4. Multiple Choice: Which of the following odors does the ester, methyl salicylate, produce? Is the odor
that of:

w) cinnamon
x) avocado
y) orange
z) oil of wintergreen

ANSWER: D -- OIL OF WINTERGREEN

5. Short Answer: What two chemical elements are found in sphalerite?

ANSWER: ZINC AND SULFUR (ZnS)

6. Short Answer: Periclase is composed of what two elements?

ANSWER: MAGNESIUM AND OXYGEN (MgO)

7. Short Answer: Who patented the common dry cell?

ANSWER: GEORGES LECLANCHÉ

8. Multiple Choice: Ca(BrO3)2 (read: C - a, left parenthesis, B - r - O - subscript 3, right parenthesis,


subscript 2) is called:

w) calcium bromate
x) calcium bromite
y) calcium dibromite
z) calcium bromide

ANSWER: W -- CALCIUM BROMATE

9. Short Answer: What is the common oxidation state of Radium?


Science Bowl Questions Chemistry - 2

ANSWER: +2 (read: Plus two)

10. Short Answer: What is the pH of a solution that is 1000 times more acidic than pure water?

ANSWER: 4

11. Multiple Choice: Which of the following are saturated aliphatic hydrocarbons?

w) alkanes
x) alkenes
y) alkynes
z) nixanes

ANSWER: W – ALKANES

12. Multiple Choice: Sucrose, ordinary table sugar, may be classified as a:

w) Monosaccharide
x) Disaccharide
y) Polysaccharide
z) Oligosaccharide

ANSWER: X -- DISACCHARIDE

13. Multiple Choice: Pure water is approximately what molar concentration:

w) .55 Molar
x) 5.5 Molar
y) 55 Molar
z) 550 Molar

ANSWER: Y -- 55 MOLAR

14. Multiple Choice: The rate at which dissolution occurs is LEAST dependent upon which one of the
following factors:

w) temperature
x) pressure
y) solution concentration
z) solute surface area

ANSWER: X -- PRESSURE

15. Short Answer: Name the person who developed a table of elements which revealed regularities in
elemental properties?

ANSWER: DMITRI MENDELEEV

16. Multiple Choice: Who was the first American chemist to receive a Nobel Prize? He was selected in
1914 for his precise determination of atomic weights.

w) Edward Frankland
x) Theodore Richards
y) John Bardeen
z) Paul Dirac

ANSWER: X -- THEODORE RICHARDS


Science Bowl Questions Chemistry - 3

17. Short Answer: What is the word that describes a substance that reacts with both strong acids and strong
bases?

ANSWER: AMPHOTERIC

18. Multiple Choice: Of the following, which has units of "Pascal".

w) force
x) volume
y) pressure
z) viscosity

ANSWER: Y -- PRESSURE

19. Short Answer: What is the name of the point on a phase diagram which indicates the temperature
above which a gas cannot be converted to a liquid?

ANSWER: THE CRITICAL TEMPERATURE

20. Short Answer: List the following atoms in order of increasing electron affinity: oxygen, boron, and
fluorine.

ANSWER: (1) BORON, (2) OXYGEN, (3) FLUORINE

21. Short Answer: What name is given to the chemical elements with atom numbers 90 to 103, inclusively.

ANSWER: ACTINIDES or ACTINIDE SERIES

22. Short Answer: The removal of two hydrogen atoms from an alcohol yie what type of chemical
substance?

ANSWER: ALDEHYDE

23. Short Answer: I will be describing a chemical element. Try to identify it with the fewest number of
clues. This chemical element is a silver-white metal, with bluish tinge, capable of taking a high polish.
T element occurs abundantly in all ordinary rocks, except limestone and sandstone; is third in
abundance of the elements in the earth's crust and used to make drink cans. The atomic number of this
element is 13. Name this element.

ANSWER: ALUMINUM

24. Multiple Choice: All alloys contain this element if they are amalga. The element is:

w) iron
x) mercury
y) gold
z) platinum

ANSWER: X -- MERCURY

25. Multiple Choice: Azo (pron: A-zo) Compounds characteristically are compounds containing the group:

w) C2
x) N2
y) O2
z) Cl2

ANSWER: X -- N2 (N = N)
Science Bowl Questions Chemistry - 4

26. Short Answer: I will be describing a chemical element. Try to identify it with the fewest number of
clues. This element is a halogen which is a liquid at standard conditions. The element volatilizes
readily at room temperature to form a reddish vapor which is very irritating to the eyes a flesh. The
atomic number of this element is 35 and its chemical symbol is Br. Name this element.

ANSWER: BROMINE

27. Short Answer: Name the 5 elements of the periodic classification wh comprise the Carbon Group.

ANSWER: CARBON, SILICON, GERMANIUM, TIN AND LEAD

28. Multiple Choice: This noncombustible liquid in its dry form is noncorrosive to common metals except
aluminum. About 90% of all of this material which is manufactured goes into the production of
chlorofluorocarbons. Is this substance:

w) carbon tetrachloride
x) methylene chloride
y) hydrogen chloride
z) methyl fluoride

ANSWER: W -- CARBON TETRACHLORIDE

29. Multiple Choice: A compound which contains two ring structures having one common carbon atom is
known as a:

w) spiro-compound
x) nonpolar compound
y) interstitial compound
z) inner compound

ANSWER: W -- SPIRO-COMPOUND

30. Multiple Choice: Which of the following elements would form an acid oxide with the formula XO2
and an acidic compound with hydrogen with the formula XH2?

w) Sodium
x) Magnesium
y) Aluminum
z) Sulfur

ANSWER: Z -- SULFUR

31. Multiple Choice: Which of the following would have the largest THIR ionization energy? Is it:

w) Boron
x) Carbon
y) Nitrogen
z) Magnesium

ANSWER: Z -- MAGNESIUM
Science Bowl Questions Chemistry - 5

32. Multiple Choice: How many atoms of oxygen are in a glucose molecule

w) 2
x) 6
y) 10
z) 12

ANSWER: X – 6

33. Multiple Choice: Which of the following compounds contains a double bond?

w) butene
x) acetylene
y) butane
z) propane

ANSWER: W -- BUTENE

34. Multiple Choice: An alcohol is related to an ester as a hydroxide compound is related to:

w) an acid
x) a ketone
y) an ether
z) a salt

ANSWER: Z -- A SALT

35. Multiple Choice: In any chemical reaction, a quantity that decrease to a minimum is:

w) free energy
x) entropy
y) temperature
z) enthalpy

ANSWER: W -- FREE ENERGY

36. Multiple Choice: It is believed that carbon-14 in nature is slowly generated by the action of:

w) protons on carbon-12
x) electrons on hydrogen
y) cosmic rays on boron
z) neutrons on nitrogen

ANSWER: Z -- NEUTRONS ON NITROGEN

37. Multiple Choice: When water at 10°C is heated to 20°C, its internal energy:

w) changes by 10°C
x) is doubled
y) is less than doubled
z) is more than doubled

ANSWER: Y -- IS LESS THAN DOUBLED

38. Multiple Choice: In the filling of electron orbitals for the element sulfur, which has 16 electrons, the
number of electrons in the 3p orbitals is:

w) 3
x) 4
Science Bowl Questions Chemistry - 6

y) 6
z) 0

ANSWER: X -- 4

39. Short Answer: You have a solution of 5 molar Sodium Phosphate and n to prepare a solution of 500
millimolar Sodium Phosphate. How much water would you add to 100 milliliter of the original 5 molar
solution to produce the 500 millimolar solution?

ANSWER: 900 MILLILITERS (or 0.9 LITERS)

40. Multiple Choice: Which of the following is the weakest acid?

w) hydrochloric acid
x) hydrofluoric acid
y) sulfuric acid
z) nitric acid

ANSWER: X -- HYDROFLUORIC ACID

41. Short Answer: Marie Curie shared the 1911 Nobel Prize in chemistry with two fellow chemists. Name
them.

ANSWER: PIERRE CURIE and (A. HENRI) BECQUEREL

42. Multiple Choice: A mystical substance supposedly released during combustion was called:

w) thermal flax
x) ether
y) caloric
z) phlogiston (pron: flow-gis-ton)

ANSWER: Z – PHLOGISTON

43. Multiple Choice: Of the following liquids, which is most dense?

w) water
x) gasoline
y) acetone
z) chloroform

ANSWER: Z -- CHLOROFORM

44. Short Answer: What is the most abundant element in the human body, WEIGHT?

ANSWER: OXYGEN

45. Multiple Choice: Solids may be considered to be either crystalline noncrystalline. The basic difference
between them is that a crystal, in contrast to a noncrystal:

w) has a sharp melting point


x) has an irregular array of atoms
y) exhibits double refraction
z) has a completely regular atomic or molecular structure

ANSWER: Z -- HAS A COMPLETELY REGULAR ATOMIC OR MOLECULAR STRUCTURE

46. Multiple Choice: The Rate Law expresses the rate of a reaction in terms of the concentrations of the:
Science Bowl Questions Chemistry - 7

w) reactants
x) products
y) by-products
z) catalysts

ANSWER: W -- REACTANTS

47. Multiple Choice: Hydrocarbons that contain a triple bond between carbon atoms are known as:

w) alkenes
x) alkanes
y) alkynes
z) polymers

ANSWER: Y – ALKYNES

48. Multiple Choice: Which of the following metals melts in your hand?

w) gallium
x) cesium
y) sodium
z) magnesium

ANSWER: W -- GALLIUM

49. Multiple Choice: When vinegar and baking soda are mixed together, w gas is formed?

w) oxygen
x) nitrogen
y) carbon dioxide
z) hydrogen

ANSWER: Y -- CARBON DIOXIDE

50. Short Answer: How many carbons are found in a propane molecule?

ANSWER: 3

51. Multiple Choice: The atoms in sugar are bound by what type of bond?

w) ionic
x) hydrogen
y) covalent
z) van der Waals

ANSWER: Y -- COVALENT

52. Multiple Choice: Which is an example of a polymer?

w) salt
x) oil
y) plastic
z) glucose

ANSWER: Y – PLASTIC

53. Multiple Choice: Vegetable oil is made into margarine through:

w) halogenation
Science Bowl Questions Chemistry - 8

x) partial hydrogenation
y) methylation
z) oxidation

ANSWER: X -- PARTIAL HYDROGENATION

54. Multiple Choice: What is the name given to the equation PV=nRT?

w) law of partial pressure


x) ideal gas equation
y) quadratic equation
z) Raoult's equation

ANSWER: X -- IDEAL GAS EQUATION

55. Short Answer: What term describes the process when a solid phase changes directly to the gas phase?

ANSWER: SUBLIMATION

56. Multiple Choice: Which of the following elements can form the great number of covalent bonds?

w) carbon
x) nitrogen
y) oxygen
z) sulfur

ANSWER: Z -- SULFUR

57. Multiple Choice: Which of the following elements forms a tetrahedra structure?

w) carbon
x) beryllium
y) boron
z) fluorine

ANSWER: W – CARBON

58. Multiple Choice: Which of the following metals is a solid at room temperature, yet melts in your hand?

w) gallium
x) cesium
y) sodium
z) magnesium

ANSWER: W -- GALLIUM

59. Short Answer: When water and an active metal react, what are the two products that result?

ANSWER: HYDROGEN AND A BASE (HYDROXIDE)


Science Bowl Questions Chemistry - 9

60. Multiple Choice: 2.5 liters of a sodium chloride (NaCl) solution contain 5 moles of the solute. What is
the molarity?

w) 5 molar
x) 2 molar
y) 2.5 molar
z) 12.5 molar

ANSWER: X -- 2 MOLAR

61. Short Answer: What is the active ingredient in many common laxative that is also an acid-base
indicator?

ANSWER: PHENOLPHTHALEIN (pron: fEn-el-thal-E-en)

62. Multiple Choice: The structure of an ammonia molecule can best be described as:

w) linear
x) tetrahedral
y) pyramidal
z) triagonal planar

ANSWER: Y – PYRAMIDAL

63. Short Answer: What naturally occurring radioactive element is so common in homes that testing for its
presence is often advisable?

ANSWER: RADON

64. Multiple Choice: How many moles of carbon are in 48 grams of carbon?

w) 4
x) 3
y) 2
z) 1

ANSWER: W -- 4

65. Short Answer: What is the molality of a salt solution prepared by dissolving two moles of salt in 500
grams of water?

ANSWER: 4

Short Answer: During electrolysis of water, what gas is evolved from the cathode?

ANSWER: HYDROGEN

66. Multiple Choice: Crude oil is separated into its components by:

w) chemical reaction
x) simple distillation
y) fractional distillation
z) settling

ANSWER: Y -- FRACTIONAL DISTILLATION

67. Short Answer: What compound occurs when ammonia and hydrogen chloride are mixed together?

ANSWER: AMMONIUM CHLORIDE FORMS


Science Bowl Questions Chemistry - 10

68. Short Answer: The acid present in vinegar is:

ANSWER: ACETIC ACID

69. Multiple Choice: What force between molecules tends to cause most common gases to behave as non-
ideal gases?

w) Van der Waals' force


x) ionic bonding force
y) gravitational force
z) covalent bonding force

ANSWER: W -- VAN DER WAALS' FORCE

70. Short Answer: What UNUSUAL type of bonding occurs in diborane, B2H6?

ANSWER: THREE CENTER BONDING

71. Multiple Choice: What is the name given to the non-superimposable mirror image forms of chiral
compounds?

w) cis-trans
x) enantiomers
y) functional isomers
z) diasteriomers

ANSWER: X -- ENANTIOMERS

72. Multiple Choice: Which of the following is the EMPIRICAL formula of dodecane?

w) C5H11
x) C6H13
y) C10H22
z) C12H26

ANSWER: X -- C6H13

73. Short Answer: What is the chemical formula for baking soda?

ANSWER: NaHCO3

74. Multiple Choice: Copper is refined commercially by a process of:

w) smelting
x) electrolysis
y) sublimation
z) roasting

ANSWER: X -- ELECTROLYSIS

75. Multiple Choice: The action of concentrated sulfuric acid on sugar best described as:

w) catalysis
x) oxidation
y) reduction
z) dehydration

ANSWER: Z -- DEHYDRATION
Science Bowl Questions Chemistry - 11

76. Multiple Choice: A metal that does NOT occur native in the earth is

w) copper
x) gold
y) sodium
z) silver

ANSWER: Y -- SODIUM

77. Short Answer: What metal is used as a reducing agent to obtain iron from iron oxide in the Thermite
process?

ANSWER: ALUMINUM

78. Short Answer: Glycerine and what other substance are made by heating animal fats or vegetable oils
with sodium hydroxide?

ANSWER: SOAP

79. Short Answer: What is the formula for the anhydride of sulfuric acid?

ANSWER: SO3

80. Multiple Choice: Under similar conditions, which of the following is the best reducing agent?

w) fluoride ion
x) chloride ion
y) bromide ion
z) iodide ion

ANSWER: Z -- IODIDE ION

81. Multiple Choice: How many carbon atoms are there in one napthalene molecule?

w) 6
x) 8
y) 10
z) 15

ANSWER: Y -- 10

82. Multiple Choice: A process in which substances are separated through differences in the rates at which
the components migrate is called:

w) filtration
x) chromatography
y) elution
z) titration

ANSWER: X -- CHROMATOGRAPHY

83. Short Answer: What is the name of the bond that joins amino acids together in a protein?

ANSWER: PEPTIDE BOND or AMIDE

84. Multiple Choice: Which bond has the LEAST ionic character?

w) H - F
Science Bowl Questions Chemistry - 12

x) Li - F
y) Li - Br
z) F - F

ANSWER: Z -- F - F

85. Multiple Choice: The lanthanide series elements are also known as:

w) rare earth elements


x) non-metals
y) fifth period elements
z) sixth period elements

ANSWER: W -- RARE EARTH ELEMENTS

86. Multiple Choice: If the first ionization energy of magnesium is 176 kilocalories per mole, one would
expect the second ionization energy t be approximately:

w) 350 kilocalories per mole


x) 1,760 kilocalories per mole
y) 200 kilocalories per mole
z) 20 kilocalories per mole

ANSWER: W -- 350 KILOCALORIES PER MOLE

87. Short Answer: When considering electron orbitals, which orbital is generally considered spherical?

ANSWER: S ORBITAL

88. Short Answer: The angle between any two carbon-hydrogen bonds in a methane molecule is how many
degrees?

ANSWER: 109.5 DEGREES

89. Short Answer: Jaroslav Heyrovsky (Czechoslovakiw) received the 1959 Nobel Prize in chemistry for
developing what technique used in chemical analysis?

ANSWER: POLAROGRAPHY

90. Multiple Choice: Ferrous is an equivalent name for which of the following?

w) Iron (IV) (pron: iron-4)


x) Iron (III) (pron: iron-3)
y) Iron (II) (pron: iron-2)
z) Iron (I) (pron: iron-1)

ANSWER: Y -- IRON (II)

91. Multiple Choice: Mass percentage of carbon in CO2 is approximately:

w) 37.5%
x) 27.3%
y) 75%
z) 45%

ANSWER: X -- 27.3%

92. Multiple Choice: Harold Clayton Urey, an American, received the 193 Nobel Prize in chemistry for his
discovery of:
Science Bowl Questions Chemistry - 13

w) Francium
x) Americium
y) deuterium
z) protium

ANSWER: Y -- DEUTERIUM

93. Multiple Choice: The most abundant element in the universe is thought to be:

w) carbon
x) oxygen
y) hydrogen
z) nitrogen

ANSWER: Y – HYDROGEN

94. Multiple Choice: The 1936 Nobel Prize in chemistry was awarded to Peter J.W. Debye for his studies
of:

w) kinetic energy of colliding particles


x) electron configuration
y) dipole moments
z) conformations of cyclohexane

ANSWER: Y -- DIPOLE MOMENTS

95. Multiple Choice: The inventor of the Kelvin temperature scale was:

w) K. Cavendish II
x) Lord William H. Harrison
y) Sir William Thomson
z) none of the above

ANSWER: Y -- SIR WILLIAM THOMSON (Lord Kelvin)

96. Multiple Choice: "Under the same conditions of temperature and pressure, the volumes of reacting
gases and of their gaseous products are expressed in ratios of small whole numbers." This law is
attributed to:

w) Dalton
x) Gay-Lussac
y) Ramsay
z) Avogadro

ANSWER: X -- GAY-LUSSAC

97. Multiple Choice: The type of crystal lattice exemplified in a diamo is:

w) ionic crystal
x) covalent network crystal
y) metallic crystal
z) covalent molecular crystal

ANSWER: X -- COVALENT NETWORK CRYSTAL

98. Multiple Choice: A crystallized substance containing water of crystallization is most appropriately
termed:
Science Bowl Questions Chemistry - 14

w) a hydrate
x) an oxide
y) a hydroxide
z) a condensation crystal

ANSWER: W -- A HYDRATE

99. Multiple Choice: The loss of water of crystallization from a hydrate when exposed to air is termed:

w) efflorescence (pron: eh-floor-es-ence)


x) deliquescence (pron: del-i-kwes-ence)
y) quiescence (pron: kwi-es-ence)
z) anhydrogenation

ANSWER: W -- EFFLORESCENCE

100. Multiple Choice: Mayonnaise is:

w) a coalescent solution
x) a hydrogenated oil
y) a covalent solution
z) an emulsion

ANSWER: Z -- AN EMULSION
Science Bowl Practice Questions Earth Science - 1

Science Bowl Practice Questions – Earth Science


1. Multiple Choice: The ice sheet that covered northeastern United States came from which of the
following locations?

w) Labrador center in eastern Canada


x) Keewatin center in central Canada
y) Cordilleran center in the Rocky Mountains
z) Greenland

ANSWER: W -- LABRADOR CENTER IN EASTERN CANADA

2. Multiple Choice: A volcano which is composed of lava flows and pyroclastic material and which is
steep-sided and very tall is known as:

w) Syncline
x) Composite cone
y) Anticline
z) none of these

ANSWER: X -- COMPOSITE CONE

3. Multiple Choice: The solidified lava of a volcano belongs to which rock family? Is it:

w) igneous
x) metamorphic
y) sedimentary
z) fossilized

ANSWER: W -- IGNEOUS

4. Multiple Choice: In geological studies, a dome shaped intrusion is called a:

w) volcanic neck
x) laccolith
y) nuee ardente
z) caldera

ANSWER: X -- LACCOLITH

5. Multiple Choice: Shiprock in New Mexico is a:

w) laccolith
x) volcanic neck
y) cinder cone
z) caldera

ANSWER: X -- VOLCANIC NECK


Science Bowl Practice Questions Earth Science - 2

6. Multiple Choice: Iceland has a great deal of volcanic activity. The reason for this is:

w) it was formed above a mid-ocean rift.


x) it is part of the "Ring of Fire".
y) two tectonic plates are rubbing against each other under Iceland.
z) the American plate is diving under the European plate in this region.

ANSWER: W -- IT WAS FORMED ABOVE A MID-OCEAN RIFT.

7. Multiple Choice: Of the following, the rock that is most resistant to both chemical and mechanical
weathering is:

w) shale
x) limestone
y) marble
z) quartzite

ANSWER: Z -- QUARTZITE

8. Short Answer: There is a specific term used by geologists to describe rock weathering in which shells
or plates are broken away from a rock leaving it rounded in appearance. What is this term?

ANSWER: EXFOLIATION

9. Short Answer: Biosphere describes the earth realm where life occurs What describes the entire solid
earth realm and is composed of mineral matter?

ANSWER: LITHOSPHERE

10. Multiple Choice: Which of the following energy sources does NOT originally come from the Sun?

w) wind
x) Ocean thermal energy conversion
y) geothermal
z) hydroelectric

ANSWER: Y – GEOTHERMAL

11. Short Answer: What is the name of the atmosphere layer of upwardly diminishing temperature situated
above the stratosphere and topped by the mesopause?

ANSWER: MESOSPHERE

12. Short Answer: What is the term used to measure the intensity of solar radiation and is equal to one
gram calorie per square centimeter per minute?

ANSWER: LANGLEY

13. Multiple Choice: What percentage of the earth's atmosphere does O2 comprise?

w) 75%
x) 50%
y) 21%
z) 32%

ANSWER: Y -- 21%

14. Short Answer: What gas makes up the majority of our atmosphere?

ANSWER: NITROGEN
Science Bowl Practice Questions Earth Science - 3

15. Multiple Choice: The tanning rays of the sun are called:

w) infrared rays
x) visible light
y) ultraviolet rays
z) gamma rays

ANSWER: Y -- ULTRAVIOLET RAYS

16. Multiple Choice: What is the relative humidity when the absolute humidity is 3 grams per cubic meter
and the air has a capacity of 12 grams per cubic meter?

w) 4%
x) 9%
y) 25%
z) 400%

ANSWER: Y -- 25%

17. Multiple Choice: While many different gases are found in the atmosphere, a number do NOT
contribute significantly to solar heating of the atmosphere. Which TWO of the following gases do NOT
absorb much heat?

w) carbon dioxide
x) nitrogen
y) water vapor
z) oxygen

ANSWER: X and Z -- NITROGEN and OXYGEN


Science Bowl Practice Questions Earth Science - 4

18. Multiple Choice: Global climate change is being attributed to the atmospheric increase in what two
gases produced by human activities?

w) oxygen and hydrogen


x) ozone and methane
y) nitrous oxide and sulfur dioxide
z) methane and carbon dioxide
e) ozone and carbon monoxide

ANSWER: Z -- METHANE AND CARBON DIOXIDE

19. Short Answer: To within 5%, how much has atmospheric CO2 concentration INCREASED since the
year 1800?

ANSWER: 25%

20. Multiple Choice: The concentration of carbon dioxide in the atmosphere is in the range of:

w) 1-10 parts per million


x) 10-100 parts per million
y) 100-1000 parts per million
z) 1000-10,000 parts per million

ANSWER: Y -- 100-1000 PARTS PER MILLION (the exact number is 350 ppm as of the time of this
article)

21. Short Answer: The ozone layer is located in which region of the atmosphere?

ANSWER: STRATOSPHERE

22. Multiple Choice: In geological studies, all of the following are TRUE of P waves except that they:

w) are body waves


x) travel like sound waves
y) can pass through liquids
z) are slower than S waves

ANSWER: Z -- ARE SLOWER THAN S WAVES

23. Multiple Choice: Which of the following terms is used to describe the appearance of a mineral in
transmitted light? Is it:

w) diaphaneity (pron: die-ah-fah-na-ah-tee)


x) translucence
y) opaqueness
z) porosity

ANSWER: W -- DIAPHANEITY

24. Multiple Choice: In geological studies, arkose contains which of the following minerals which gives it
a reddish color?

w) orthoclase
x) calcite
y) aragonite (pron: a-rag-o-nite)
z) quartz

ANSWER: W -- ORTHOCLASE

25. Multiple Choice: Which one of the following minerals does NOT contain silica tetrahedrons?
Science Bowl Practice Questions Earth Science - 5

w) quartz
x) muscovite
y) halite
z) orthoclase

ANSWER: Y – HALITE

26. Multiple Choice: Which of the following minerals is noted for its one perfect cleavage?

w) calcite
x) muscovite
y) quartz
z) pyrite

ANSWER: X -- MUSCOVITE

27. Multiple Choice: Which of the following gases is given off in the acid test of a carbonate mineral?

w) chlorine
x) nitrogen
y) carbon dioxide
z) sulfur dioxide

ANSWER: Y -- CARBON DIOXIDE

28. Multiple Choice: Fine parallel lines, or striations, are most likely to be seen on a cleavage surface of
which of the following minerals?

w) quartz
x) orthoclase
y) hornblende
z) plagioclase (pron: play-gee-o-klase)

ANSWER: Z – PLAGIOCLASE
Science Bowl Practice Questions Earth Science - 6

29. Multiple Choice: Which of the following minerals is NOT a silicate?

w) orthoclase
x) muscovite
y) augite (pron: aw-jIt)
z) magnetite

ANSWER: Z – MAGNETITE

30. Multiple Choice: Which of the following pairs contains one igneous and one sedimentary rock?

w) shale and marble


x) sandstone and quartzite
y) granite and limestone
z) obsidian and gneiss

ANSWER: Y -- GRANITE AND LIMESTONE

31. Multiple Choice: Which of these pairs of minerals are always found in granites?

w) muscovite and calcite


x) quartz and orthoclase
y) hornblende and talc
z) augite and magnetite

ANSWER: X -- QUARTZ AND ORTHOCLASE

32. Multiple Choice: An example of a rock whose minerals have been crushed into thin sheets or bands is:

w) shale
x) schist
y) conglomerate
z) granite

ANSWER: X -- SCHIST

33. Multiple Choice: The hydrolysis of orthoclase results in the formation of:

w) shale
x) kaolin
y) lime
z) hydrochloric acid

ANSWER: X – KAOLIN
Science Bowl Practice Questions Earth Science - 7

34. Multiple Choice: The deposits of loess in the United States probably originated as which of the
following?

w) talus
x) rock flour
y) humus
z) geyserite

ANSWER: X -- ROCK FLOUR

35. Multiple Choice: This sedimentary rock was used to build the Pantheon and the Colossium:

w) marble
x) travertine
y) sandstone
z) ironstone

ANSWER: X -- TRAVERTINE

36. Short Answer: What is another name for the semi-precious stone heliotrope?

ANSWER: BLOODSTONE

37. Short Answer: Rocks are classified depending upon how they are formed. In which class would slate
be found?

ANSWER: METAMORPHIC

38. Multiple Choice: Which of the following is an igneous rock?

w) Granite
x) Shale
y) Coal
z) Slate

ANSWER: W – GRANITE

39. Multiple Choice: Which of the following is a metamorphic rock?

w) sandstone
x) granite
y) quartz
z) marble

ANSWER: Z – MARBLE
Science Bowl Practice Questions Earth Science - 8

40. Multiple Choice: The black sand of the Hawaiian Islands is composed of which of the following?

w) dark limestone
x) quartz
y) gypsum
z) basalt

ANSWER: Z -- BASALT

41. Short Answer: Name the clay mineral produced by the alteration of potash feldspar which is also used
in ceramics.

ANSWER: KAOLINITE

42. Multiple Choice: All of the following are examples of chemically precipitated sedimentary rock
except:

w) limestone
x) sandstone
y) dolomite

ANSWER: X -- SANDSTONE

43. Short Answer: Name the extremely fine-grained, wind-blown clay particularly characteristic of the arid
and semi-arid southwestern United States, Mexico and South America. This material was used by the
southwestern Indians and Mexicans for constructing huts and buildings from prehistoric times.

ANSWER: ADOBE

44. Multiple Choice: Which of the following materials is the hardest?

w) calcite
x) silicon carbide
y) topaz
z) quartz

ANSWER: X -- SILICON CARBIDE

45. Multiple Choice: Igneous rocks that form from magma are known as:

w) minerals
x) granite
y) intrusive rocks
z) gneiss (pron: nice)

ANSWER: Y -- INTRUSIVE ROCKS

46. Multiple Choice: The mineral halite is:

w) potassium chloride
x) sodium chloride
y) calcium chloride
z) calcium bromide

ANSWER: X -- SODIUM CHLORIDE

47. Multiple Choice: What is the name of the white clay which has been used for thousands of years in the
fabrication of ceramic bodies? Is it:

w) Talc
Science Bowl Practice Questions Earth Science - 9

x) Kaolin
y) Feldspar
z) Quartz

ANSWER: X -- KAOLIN

48. Multiple Choice: An agate is a form of what mineral?

w) magnetite
x) feldspar
y) quartz
z) copper

ANSWER: Y – QUARTZ

49. Multiple Choice: Which of the following is the name of the partly melted rock layer on which the
plates move?

w) lithosphere
x) asthenosphere
y) hydrosphere
z) outer core

ANSWER: X -- ASTHENOSPHERE

50. Multiple Choice: Which of the following areas is closest to the total surface area of the earth in square
kilometers?

w) 500 million
x) 500 billion
y) 500 thousand
z) 5 billion

ANSWER: W -- 500 MILLION


Science Bowl Practice Questions Earth Science - 10

51. Multiple Choice: We know that the OUTER core is liquid because:

w) P waves pass through it


x) S waves pass through it
y) P waves cannot pass through it
z) S waves cannot pass through it

ANSWER: Z -- S WAVES CANNOT PASS THROUGH IT

52. Short Answer: It is 9:00 p.m. in Greenwich, England. You are on a ship whose local time is 6:00 p.m.
What is the longitude of your ship?

ANSWER: 45 DEGREES WEST

53. Multiple Choice: Field capacity represents the maximum quantity of water held:

w) in the soil against the force of gravity


x) in the ground water zone
y) in the intermediate zone
z) below the water table

ANSWER: W -- IN THE SOIL AGAINST THE FORCE OF GRAVITY

54. Multiple Choice: Trade winds are persistent easterly winds develop between:

w) subtropical high pressure and equatorial doldrum


x) subtropical high pressure and subpolar low pressure
y) westerlies and polar highs
z) none of the above

ANSWER: W -- SUBTROPICAL HIGH PRESSURE AND EQUATORIAL DOLDRUM

55. Multiple Choice: An open-ocean community is also called a:

w) pelagic community (pron: pe-laj-ick)


x) benthic community (pron: ben-thik)
y) lentic community (pron: len-tick)
z) lotic community (pron: lot-ick)

ANSWER: W -- PELAGIC COMMUNITY

56. Multiple Choice: A lake or pond community is also known as a:

w) pelagic community (pron: pe-laj-ick)


x) benthic community (pron: ben-thick)
y) lentic community (pron: len-tick)
z) lotic community (pron: lot-ick)

ANSWER: Y -- LENTIC COMMUNITY

57. Short Answer: What is the name given to a region where one plate of the earth's crust is sliding under
another?

ANSWER: SUBDUCTION ZONE

58. Multiple Choice: The tectonic plates on which India and the adjacent portion of Asia ride are colliding
at a rate of:

w) 1-10 millimeters per year


x) 1-10 centimeters per year
Science Bowl Practice Questions Earth Science - 11

y) 1-10 meters per year


z) 1-10 decimeters per year

ANSWER: X -- 1-10 CENTIMETERS PER YEAR

59. Multiple Choice: In which of these states can large numbers of drumlins be found?

w) Oregon
x) California
y) Texas
z) Wisconsin

ANSWER: Z -- WISCONSIN

60. Multiple Choice: Which one of these glacial features is believed to have formed in the bed of a
subglacial stream?

w) an esker
x) a kame
y) a drumlin
z) a moraine

ANSWER: W -- AN ESKER

61. Multiple Choice: In which of the following is a continental glacier found?

w) Switzerland
x) the Rockies
y) Greenland
z) the Andes Mountains

ANSWER: Y – GREENLAND
Science Bowl Practice Questions Earth Science - 12

62. Multiple Choice: Deposits that form when the ice in a glacier melts are called:

w) drifts
x) dunes
y) veins
z) cirques (pron: serks)

ANSWER: W – DRIFTS

63. Multiple Choice: The earth's climate has been warming since the last glacial period which ended about
how many years ago?

w) 1,000 to 2,000 years ago


x) 10,000 to 20,000 years ago
y) 100,000 to 200,000 years ago
z) 1,000,000 to 2,000,000 years ago

ANSWER: X -- 10,000 TO 20,000 YEARS AGO

64. Multiple Choice: Cirrus clouds are:

w) rain clouds
x) thick and fleecy
y) made of ice crystals
z) low cloud layers

ANSWER: Y -- MADE OF ICE CRYSTALS

65. Multiple Choice: The name of a rapid rise of coastal water that accompanies the arrival of a cyclone is
called:

w) hurricane
x) storm surge
y) tidal wave
z) cyclonistat

ANSWER: X -- STORM SURGE

66. Multiple Choice: Certain substances are classified as water pollutants, others as air pollutants and still
others as land pollutants. Which of the following pollutes all three parts of our environment?

w) fluorocarbons
x) smog
y) acid rain
z) ozone

ANSWER: Y -- ACID RAIN

67. Short Answer: What is the name of the phenomenon that causes an accumulation of heat in the lower
atmosphere because of the absorption of long-wave radiation from the Earth's surface?

ANSWER: GREENHOUSE EFFECT

68. Multiple Choice: Which is true for Low-Latitude Climates?

w) have a winter season


x) lack a winter season
y) are wet throughout the year
z) are dry throughout the year
Science Bowl Practice Questions Earth Science - 13

ANSWER: X -- LACK A WINTER SEASON

69. Multiple Choice: The Greenhouse Effect is caused primarily by which of the following processes?

w) blocking incoming short wave solar energy


x) absorbing incoming short wave solar energy
y) heating of the Earth's stratosphere
z) absorbing outgoing long wave radiation from the Earth

ANSWER: Z -- ABSORBING OUTGOING LONG WAVE RADIATION FROM THE


EARTH

70. Multiple Choice: Fluorite is what number on Moh's hardness scale?

w) 1
x) 2
y) 3
z) 4

ANSWER: Z -- 4

71. Multiple Choice: Apatite is what number on Moh's (pron: moes: rhyme with toes) hardness scale?

w) 1
x) 3
y) 5
z) 7

ANSWER: Y – 5
Science Bowl Practice Questions Earth Science - 14

72. Multiple Choice: Hematite is a primary ore of which of the following metals?

w) zinc
x) tungsten
y) iron
z) none of the above

ANSWER: Y -- IRON

73. Multiple Choice: Galena is a primary ore of which of the following metals?

w) zinc
x) tungsten
y) lead
z) silver

ANSWER: Y -- LEAD

74. Multiple Choice: Sphalerite is a primary ore of which of the following metals?

w) zinc
x) tin
y) copper
z) iron

ANSWER: W -- ZINC

75. Multiple Choice: Which of the following minerals is a source of copper?

w) Bauxite (pron: box-ite)


x) Hematite
y) Malachite
z) Galena

ANSWER: Y -- MALACHITE

76. Multiple Choice: Which physical phenomenon or principle is most often applied in the search for
petroleum reservoirs?

w) gravitational attraction
x) magnetic field distortions
y) natural radioactive decay of minerals
z) acoustic wave transmission and reflection

ANSWER: Z -- ACOUSTIC WAVE TRANSMISSION AND REFLECTION


Science Bowl Practice Questions Earth Science - 15

77. Multiple Choice: Which of the following time periods describes the age of the coal deposits of the
Eastern United States?

w) Jurassic and Cambrian


x) Mississippian and Pennsylvanian
y) Pliocene and Miocene
z) none of these

ANSWER: X -- MISSISSIPPIAN AND PENNSYLVANIAN

78. Multiple Choice: Of the following four geologic periods, which is the oldest?

w) Devonian
x) Silurian
y) Permian
z) Jurassic

ANSWER: X -- SILURIAN

79. Multiple Choice: Of the following four geologic periods, which is the most recent?

w) Devonian
x) Silurian
y) Permian
z) Jurassic

ANSWER: Z -- JURASSIC

80. Multiple Choice: On the Phanerozoic (pron: fan-eh-reh-zO-ik) Time Scale, which of the following
geologic periods occurred approximately 225 to 280 million years ago? Is it:

w) Jurassic
x) Permian
y) Carboniferous
z) Silurian

ANSWER: X – PERMIAN

81. Multiple Choice: On the Phanerozoic (pron: fan-eh-row-zo-ick) Time Scale, which of the following
geologic periods occurred approximately 395 to 430 million years ago? Is it:

w) Jurassic (pron: ju-ras-ick)


x) Permian
y) Carboniferous
z) Silurian

ANSWER: Z -- SILURIAN

82. Multiple Choice: On the Phanerozoic (pron: fan-eh-reh-zO-ik) time scale, which of the following
geologic periods occurred approximately 500 to 570 million years ago? Is it:

w) Cambrian
x) Ordovician (pron: ord-eh-vish-en)
y) Carboniferous
z) Silurian

ANSWER: W -- CAMBRIAN

83. Multiple Choice: The Miocene, Eocene and Paleocene epoches belong to what geologic period on the
Phanerozoic Time Scale? Is it the:
Science Bowl Practice Questions Earth Science - 16

w) Devonian
x) Permian
y) Triassic (pron: try-as-ick)
z) Tertiary

ANSWER: Z -- TERTIARY

84. Multiple Choice: During which period, in the earth's history, did MOST dinosaurs become extinct?

w) Carboniferous
x) Cretaceous
y) Quaternary
z) Triassic

ANSWER: X – CRETACEOUS

85. Multiple Choice: The disappearance of the dinosaurs also marks the end of which geological era?

w) Precambrian
x) Cambrian
y) Mesozoic (pron: mez-ah-zo-ik)
z) Cenozoic (pron: see-nah-zo-ik)

ANSWER: Y -- MESOZOIC

86. Multiple Choice: On a topographic map, lines that indicate water depth are termed:

w) isogonic lines (pron: eye-seh-gone-ik)


x) bathymetric lines (pron: bath-i-meh-trik)
y) isopachs
z) none of these

ANSWER: X -- BATHYMETRIC LINES

87. Multiple Choice: In geological studies, a fault in the earth's surface in which the hanging wall has
moved UP relative to the footwall is termed:

w) overthrust fault
x) reverse fault
y) tear fault
z) none of these

ANSWER: X -- REVERSE FAULT

88. Multiple Choice: In geological studies, a fault in the earth's surface along which primarily horizontal
movement has taken place is termed:

w) overthrust fault
x) strike-slip fault
y) reverse fault
z) none of the above

ANSWER: X -- STRIKE-SLIP FAULT

89. Multiple Choice: In studies of the surface of the earth, a reverse fault having a dip between 10 and 45
degrees is known as a:

w) thrust fault
x) normal fault
y) strike-slip fault
Science Bowl Practice Questions Earth Science - 17

z) neither of these

ANSWER: W -- THRUST FAULT

90. Short Answer: What is the name given to a flat-topped plateau bordered on all sides by cliffs?

ANSWER: MESA or BUTTE

91. Multiple Choice: You are adrift off the coast of Miami, Florida in the Gulf Stream. Where would you
most likely land?

w) Argentina
x) Louisiana
y) New York
z) The British Isles

ANSWER: Z -- THE BRITISH ISLES

92. Short Answer: A troughlike downfold of strata is called a:

ANSWER: SYNCLINE

93. Multiple Choice: The horse latitude which occurs at approximately 30o - 35o north is also known as
the:

w) doldrums
x) calms of Cancer
y) calms of Capricorn
z) trough of the Pacific

ANSWER: X -- CALMS OF CANCER

94. Short Answer: Of the three types of corals, Scleractinian, Rugose and Tabulate, which one is still in
existence today?

ANSWER: SCLERACTINIAN

95. Multiple Choice: A circular cavity ground out of bedrock by running water is called a:

w) cave
x) pothole
y) sinkhole
z) kettle hole

ANSWER: X -- POTHOLE

96. Multiple Choice: A yazoo stream flows through which of the following?

w) gorge
x) divide
y) backswamp
z) water gap

ANSWER: Y -- BACKSWAMP

97. Multiple Choice: Which of the following is a famous hanging trough waterfall?

w) Yellowstone Falls
x) Yosemite Falls
y) Niagara Falls
z) Victoria Falls
Science Bowl Practice Questions Earth Science - 18

ANSWER: X -- YOSEMITE FALLS


Science Bowl Practice Questions Earth Science - 19

98. Multiple Choice: A wave with a wavelength of 40 meters and a height of 2 meters will NOT be
refracted in water with a depth of more than:

w) 1 meter
x) 2 meters
y) 10 meters
z) 20 meters

ANSWER: Z -- 20 METERS

99. Multiple Choice: The lag time between precipitation period and peak discharge period of a river:

w) is increased by urbanization
x) is decreased by urbanization
y) is shorter for a large drainage basin than for a small one
z) depends upon the hydraulic heads

ANSWER: X -- IS DECREASED BY URBANIZATION

100. Short Answer: Give the general term used to designate the sand, silt, and mud deposited by a
stream, along its bank or upon its floodplain, during periods of high water.

ANSWER: ALLUVIUM or ALLUVIAL


Science Bowl Practice Questions Math - 1

Science Bowl Practice Questions – Math


1. Short Answer: What is the volume of a sphere of radius "R"?

ANSWER: (4/3) p R3

2. Short Answer: What is the surface area of a sphere of radius "r"?

ANSWER: 4 p R2 or 12.566 x R2

3. Short Answer: How many degrees are there in p radians?

ANSWER: 180 DEGREES

4. Short Answer: Using an x-y coordinate axis, the figure represented by the equation [x2/36] + [y2/16] =
1 is centered about what x-y coordinate point?

ANSWER: (0,0)

5. Short Answer: An ellipse is represented by the equation [x2/36] + [y2/16] = 1. What is the length of the
major axis of this ellipse?

ANSWER: 12

6. Short Answer: Using an x-y coordinate axis, a parabola is given by the equation y = x2. Give the x-y
coordinates of the focal point for this parabola.

ANSWER: (0,1/4)

7. Short Answer: Using an x-y coordinate axis, a parabola is represented by the equation x2 = 6y. The
vertex of this parabola is at what coordinate point?

ANSWER: (0,0) or x = 0, y = 0

8. Short Answer: For a right triangle, the sin(A) is 3/5. To what value is the tan(A) (read: tangent of A)
equal?

ANSWER: ¾

9. Multiple Choice: The tan(-A) (read: tangent of the angle minus A) is equal to which of the following?

w) tan(A) (read: tangent of the angle A)


x) -tan(-A) (read: minus the tangent of the angle minus A)
y) -tan(A) (read: minus the tangent of the angle A)
z) none of the above

ANSWER: Y -- -TAN(A) (read: minus the tangent of the angle A)


Science Bowl Practice Questions Math - 2

10. Multiple Choice: The cotangent of the angle A is equal to which of the following?

w) cot(-A) (pron: cotangent of the angle minus A)


x) -cot(-A) (pron: minus the cotangent of the angle minus A)
y) -cot(A) (pron: minus the cotangent of the angle A)
z) none of the above

ANSWER: X -- -COT(-A)

11. Multiple Choice: The tan(180o + A) (read: tangent of the quantity 18 degrees plus A) is equal to which
of the following?

w) tan(A)
x) -tan(A)
y) tan(-A)
z) none of the above.

ANSWER: W -- TAN(A)

12. Multiple Choice: The sin(2A) (read: the sine of angle 2A) is equal to which of the following
relationships?

w) sin(A) - cos(A) (read: sine of A minus the cosine of A)


x) 3sin(A) / cos(A) (read: 3 times the sine of A divided by the cosine of A)
y) 2 sin(A) cos(A) (read: 2 times the sine of A times the cosine of A)
z) tan(A) - 1 (read: tangent of A minus 1)

ANSWER: Y -- 2 TIMES THE SIN(A) TIMES THE COS(A)

13. Multiple Choice: The sin2(A/2) (read: sine squared of angle A divided by 2) is equal to which of the
following relationships? Is it:

w) (1/2)[1 - cos(A)] (read: 1/2 times the quantity one minus the cosine of the angle A)
x) (1/2)[1 - sin(A)] (read: 1/2 times the quantity one minus the sine of the angle A)
y) 2 sin(A) cos(A)
z) none of the above

ANSWER: W -- (1/2)[1 - cos(A)]

14. Short Answer: For what value of the angle "A" is the cos(A) equal t the sin(A)?

ANSWER: 45o or p/4 RADIANS or (45o + n times 180o)

15. Short Answer: Given that the log[base 10](3) = .477, what is the log[base 10] of 300?

ANSWER: 2.477
Science Bowl Practice Questions Math - 3

16. Short Answer: What is the log[base 5](1/25) equal to?

ANSWER: -2

17. Short Answer: Find x if log[base 5](x) = -2.

ANSWER: 1/25 or .04

18. Short Answer: Given that the log[base 10](2) = .301, what is the log[base 10](8) equal to?

ANSWER: .903

19. Short Answer: Given that the log[base 10](2) = .30 and that the log[base 10](3) =.48, what is the
log[base 10](1.5) equal to?

ANSWER: .18

20. Multiple Choice: The trigonometric function cos(2A) (read: cosine of the angle 2A) is equal to which
of the following relationships?

w) 1 - [sin(A)]2 (read: one minus the sine squared of A)


x) [cos(A)]2 - [sin(A)]2 (read: cosine squared of angle A minus the sine squared of
angle A)
y) 2[sin(A)][cos(A)] (read: 2 times the sine of A times the cosine of A)
z) none of the above

ANSWER: X -- [cos(A)]2 - [sin(A)]2

21. Short Answer: Find the equation of the line that passes through the point (1,2) and is perpendicular to
the line y = 2x + 1.

ANSWER: Y = -(1/2)x + 2.5 or y - 2 = (-1/2)(x - 1)

22. Multiple Choice: A "reunion of broken parts" is the meaning of the root of what word:

w) algebra
x) calculus
y) mathematics
z) none of the above

ANSWER: W -- ALGEBRA

23. Short Answer: In what number base does 1 + 2 = 10?

ANSWER: BASE 3

24. Short Answer: How is the number 14 represented in the hexadecimal system?

ANSWER: E

25. Short Answer: If an electronic circuit operates in 100 nanoseconds, how many operations can it
perform in one second?

ANSWER: 10 MILLION or 107

26. Short Answer: What is the imaginary unit "i" when raised to the power 10?

ANSWER: -1
Science Bowl Practice Questions Math - 4

27. Short Answer: If both x and y are functions of time, what is the derivative of the PRODUCT of x and y
with respect to time equal to?

ANSWER: x[dy/dt] + y[dx/dt] or y[dx/dt] + x[dy/dt]

28. Short Answer: If x = t4 (read: t raised to the fourth), what is the second derivative of x with respect to t
equal to?

ANSWER: 12t2 (read: 12t squared)

29. Short Answer: In calculus, what is the indefinite integral of the sin(U)dU equal to?

ANSWER: -cos(U) + CONSTANT

30. Short Answer: Two off-axis lines are perpendicular if and only if the product of their slopes is equal to
what?

ANSWER: -1

31. Short Answer: Give the equation of one of the circles which satisfy the following criteria: a circle with
a radius of 8 which is tangent to the y-axis and whose center lies on the x-axis.

ANSWER: (x - 8)2 + (y)2 = 82 or (x + 8)2 + (y)2 = 82

32. Short Answer: What is the sum of the first 100 positive integers?

ANSWER: 5050

33. Short Answer: What is the name given to the radius of a circle that can be inscribed in any regular
polygon?

ANSWER: APOTHEM

34. Short Answer: The universal set U contains the elements {1,2,3,4,5}. Set A contains the elements
{1,3}. What elements does the set A' (read: A prime) contain?

ANSWER: {2,4,5}
Science Bowl Practice Questions Math - 5

35. Short Answer: Two fair coins are tossed and at least one is a head. What is the probability that both
coins are heads?

ANSWER: 1/3

36. Multiple Choice: The polar equation, r = 3sin(3t), (read: r equals 3 times the sine of angle 3t) represents
which of the following graphs? Is it the graph of a:

w) cardioid (pron: car-dee-oiz)


x) three-leaved rose
y) Archimedian-spiral
z) limacon (pron: lee-ma-son)

ANSWER: X -- THREE-LEAVED ROSE

37. Multiple Choice: The polar equation, r = 3 - 2cos(t), (read: r equal 3 minus 2 times the cosine of the
angle t) represents which of the following graphs? Is it the graph of a:

w) cardioid
x) three-leaved rose
y) Archimedian-spiral
z) limacon (pron: lee-ma-son)

ANSWER: Z -- LIMACON

38. Short Answer: The ratio of 2 positive numbers is 3 to 8 and their product is 864.
What are the two numbers?

ANSWER: 18,48

39. Short Answer: Name the famous mathematician/physicist who was born in the year Galileo died.

ANSWER: (ISAAC) NEWTON

40. Short Answer: A wheel is rolling without slipping on a flat surface. If the diameter of the wheel is 1
meter, how far does the hub of the wheel move when the wheel turns once?

ANSWER: p METERS or 3.14 METERS

41. Short Answer: Exactly how many of the prime numbers are even?

ANSWER: 1

42. Short Answer: Give two trigonometric functions which are NEGATIVE in BOTH the third and fourth
quadrants.

ANSWER: SINE and COSECANT

43. Short Answer: An 8 meter length of rope is placed along the circumference of a circle whose diameter
is 16 meters. At the circle's center, how many radians are subtended by the 8 meter length of rope?

ANSWER: 1

44. Short Answer: How many vertices does a tetrahedron have?

ANSWER: FOUR

45. Short Answer: What term is used to describe a triangle that has three UNEQUAL sides?

ANSWER: SCALENE
Science Bowl Practice Questions Math - 6

46. Short Answer: What is the slope of the straight line whose x-intercept is +4 (read: plus 4) and whose y-
intercept is +8 (read: plus 8)?

ANSWER: -2 (read: minus 2)

47. Short Answer: The relation between y and x is given by the equation " y = ABS(4 -x) " (read: y equals
the absolute value of the quantity 4 minus x). If y equals 6, give all possible values for X.

ANSWER: 10 AND -2

48. Short Answer: The relationship between y and x is given by the equation, y = x2 + 7x + 14 (read: y
equals x squared plus 7 x plus 14). For what values of x will y equal +2?

ANSWER: -4 AND –3

49. Short Answer: The relationship between y and x is given by the equation: y = x2 - 2x + 8. For what
value of x is y a minimum?

ANSWER: 1

50. Short Answer: A straight line is plotted on an XY coordinate axis. The line intercepts the y axis at 3
and makes an angle of 45 degrees with respect to the x axis. Give the equation for this line.

ANSWER: y = x + 3 or y = -x + 3

51. Short Answer: A straight line intersects a circle in the first and second quadrants for x values of +3 and
-3. The equation of the circle is " x2 + y2 = 25 ". What is the EQUATION of the straight line?

ANSWER: y = 4
Science Bowl Practice Questions Math - 7

52. Short Answer: Twelve socks are in a drawer; 4 are red, 7 are blue, and 1 is green. What is the
maximum number of socks that you can pull out and only have 2 matching pairs?

ANSWER: 7

53. Short Answer: The scale on a blueprint of a bicycle says one half inch equals 1 foot. What will be the
actual DIAMETER of the bicycle wheel if they are drawn with a RADIUS of ¾ inches.

ANSWER: 3 FEET (accept 36 inches)

54. Short Answer: What is the value of 125 to the two-thirds power?

ANSWER: 25

55. Short Answer: What is the value of the integral from 1 to e of du/u (read: d u divided by u), where e is
the base of the natural logarithm?

ANSWER: 1

56. Short Answer: What is the slope of the curve y = x3 - 6x + 2 when it crosses the y axis?

ANSWER: -6

57. Short Answer: What is the area bounded by the curve y = x2 and the axis between x = 1 and x = 2?

ANSWER: 7/3 (or 2 and 1/3 or 2.333)

58. Short Answer: What is the slope of the curve y = ln x (read: y equals the natural logarithm of x) at x =
2?

ANSWER: 1/2

59. Short Answer: Name the two discoverers of the fundamental theorem of calculus. Last names only are
acceptable.

ANSWER: (SIR ISAAC) NEWTON AND (BARON VON) LEIBNITZ

60. Multiple Choice: The equation: [(x - 2)2/6] – [(y - 3)2/5] = 2 defines which of the following? (read: the
quantity x minus 2 squared divided by 6 MINUS the quantity y minus 3 squared divided by 5 equals 2)

w) ellipse
x) hyperbola
y) spiral
z) parabola

ANSWER: X -- HYPERBOLA

61. Short Answer: What is the distance between the points P and Q if their cartesian coordinates are (5, 5,
1) and (1, 5, -2)? (read: 5 comma, 5 comma, 1 and 1 comma, 5 comma, minus 2)

ANSWER: 5

62. Short Answer: What are the cartesian coordinates of the point of intersection of the two lines x - y = 5
and x + y = 1? (read: x minus y equals 5 and x plus y equals 1)

ANSWER: (3, -2) (accept x = 3 and y = -2 but not x = -2, y = 3)

63. Short Answer: What is the slope of the line 2x + 3y = 7? (read: 2 x plus 3 y equals 7)

ANSWER: - 2/3 or -0.667


Science Bowl Practice Questions Math - 8

64. Short Answer: Express the product of the following complex numbers in the form a + bi (read: a plus b
i) where a and b are real: 4 + 2i and 2 + 5i (read: 4 plus 2 i and 2 plus 5 i)

ANSWER: -2 + 24i (read: minus 2 plus 24 i)

65. Short Answer: In the complex plane, what are the roots of the equation x2 + 4x + 8 = 0? (read: x
squared plus 4 x plus 8 equals 0)

ANSWER: -2 - 2i and -2 + 2i

66. Multiple Choice: If logx81 = 6 (read: the log base x of 81 equals 6), then x is equal to:

w) 3
x) 9
y) 1/3
z) 91/3 (read: the cube root of 9)

ANSWER: Z -- 91/3 (read: the cube root of 9)

67. Multiple Choice; Matrix C is the product of matrix A times matrix B Which of the following must be
TRUE.

w) The number of rows of A equals the number of rows of B.


x) The number of columns of A equals the number of columns of B.
y) The number of rows of A equals the number of columns of B.
z) The number of columns of A equals the number of rows of B.

ANSWER: Z -- THE NUMBER OF COLUMNS OF A EQUALS THE NUMBER OF


ROWS

68. Short Answer: For which values of X from the set of real numbers does the following inequality hold:
8x - 6 < 2x + 6 (read: 8 x minus 6 is less than 2 x plus 6)

ANSWER: X < 2 (read: X less than 2)

69. Short Answer: To what value does the infinite series 1/4 + 1/8 + 1/16 + . . . ..converge? (read: one
fourth plus one eighth plus one sixteenth, etc.)

ANSWER: 1/2

70. Short Answer: If (sin(3x) + cos(3x))2 = 2 (read: the quantity sine x plus cosine 3 x squared equals 2)
then what does 3sin(3x) cos(3x) (read 3 sine 3 x cosine 3 x) equal?

ANSWER: 3/2

71. Short Answer: If θ = 30°, what is csc2 θ - cot2 θ equal to? (read: If theta equals 30 degrees, what is
cosecant squared theta minus cotangent squared theta equal to?)

ANSWER: 1

72. Short Answer: A woman is 22 years older than her daughter. Nine years ago she was twice as old as
her daughter. What are their present ages?

ANSWER: THE WOMAN IS 53 YEARS OLD, THE DAUGHTER IS 31 YEARS OLD.

73. Short Answer: What is the period of the function t = cos 4s? (read: t equals the cosine of 4 s)

ANSWER: p /2 (or 1/2 p)


Science Bowl Practice Questions Math - 9

74. Short Answer: What is the amplitude of the curve y = -1.5 sin x (read: y equals minus 1 point 5 sine x)?

ANSWER: 1.5 (do NOT accept -1.5)

75. Multiple Choice: Which of the following functions is NOT a transcendental function? Is it:

w) trigonometric
x) algebraic
y) logarithmic
z) exponential

ANSWER: X -- ALGEBRAIC

76. Short Answer: y = 3x2 + 4x + 2 (read: y equals 3 x squared plus 4 x plus 2). What is the value of the
derivative of y at x = 1?

ANSWER: 10

77. Short Answer: y = 4x + 3. What is the value of the integral of y from x = 0 to x = 1? (read: y equals 4 x
plus 3)

ANSWER: 5
Science Bowl Practice Questions Math - 10

78. Short Answer: A circle with a radius of 2 is defined by the equation x2 + y2 - 2y = 3. What are the
coordinates in the cartesian coordinate system of the center of this circle?

ANSWER: 0,1 or x = 0, y = 1

79. Short Answer: The polar coordinates of a point are (3, p/2) (read: 3 comma p over 2). What are the
cartesian coordinates of this point?

ANSWER: 0,3 or x = 0, y = 3

80. Short Answer: The area bounded by the race track at your high school is a rectangle with semicircles at
each end. The radius of the circular parts is 30 yards and the perimeter of the area bounded by the track
is 40 yards. Within ten yards, how long is each straight section of the track?

ANSWER: 106 YARDS (accept 96 to 116)

81. Short Answer: The population of a near-by town was 10,000 in January, 1990. It is estimated that the
population will increase by 10% each year. What will be population of the town in January, 1992.

ANSWER: 12,100

82. Multiple Choice: A fractal is a type of:

w) fraction
x) geometric shape
y) geologic fault
z) differential equation

ANSWER: X -- GEOMETRIC SHAPE

83. Short Answer: For which values of x from the set of real numbers does the following inequality hold:
4x + 5 < x + 11 (read: 4 x plus 5 is less than x plus 11)

ANSWER: x < 2 (read: x LESS THAN 2)

84. Short Answer: If two sides of a quadrilateral are parallel and the other two sides are equal but NOT
parallel, the quadrilateral is known as a:

w) parallelogram
x) triangle
y) square
z) trapezoid

ANSWER: Z – TRAPEZOID
Science Bowl Practice Questions Math - 11

85. Short Answer: A student finds that the numerical values for the are and the volume of a sphere are
equal. What is the radius for this sphere?

ANSWER: 3 (no units specified)

86. Short Answer: A customer purchased a candy bar costing $0.78 and was 3 cents short of having the
exact amount without breaking a $1 bill. How much change did the customer have after the $1 bill was
broken?

ANSWER: $0.97

87. Short Answer: A number minus its reciprocal is 14 larger than the original number. What was the
number?

ANSWER: - 1/14

88. Short Answer: A curve represented by the equation y = - 5 x2 intersects the line y = -20 at two points.
What is the straight-line distance between the two intersections?

ANSWER: 4

89. Short Answer: Solve this indefinite integral: [2/x] dx (read: the integral of 2 over x, dx).

ANSWER: 2 ln x + CONSTANT

90. Short Answer: What is the volume of a right circular cone with radius r and height h?

ANSWER: (1/3)pr2h

91. Short Answer: What is 811/4 (read: 81 to the one-fourth power)?

ANSWER: 3

92. Short Answer: Where does the graph of cotangent of x cross the x-axis?

ANSWER: x = p/2, 3p/2, 5p/2 ... or ODD INTEGERS OF p/2

93. Short Answer: A biker goes EAST for 4 miles, NORTH for 12, EAST for another 4 and SOUTH for 6.
What is the magnitude of the displacement?

ANSWER: 10 MILES
Science Bowl Practice Questions Math - 12

94. Multiple Choice: What type of numbers are included in the real number line that are not included in the
rational number line?

w) whole numbers
x) integers
y) irrational numbers
z) fractional numbers

ANSWER: Y -- IRRATIONAL NUMBERS

95. Multiple Choice: A square matrix which has all zeros below the main diagonal is called:

w) upper triangular
x) singular
y) lower triangular
z) invertable

ANSWER: W -- UPPER TRIANGULAR

96. Short Answer: Express the binary number 10010 as a decimal number.

ANSWER: 18

97. Short Answer: Express the decimal number 27 in Base 16 (Hexadecimal).

ANSWER: 1B

98. Short Answer: What is the limit of the cos(X) (read: cosine of X) a X approaches zero?

ANSWER: 1

99. Short Answer: What is the limit of the fraction 1/x as x approaches infinity?

ANSWER: 0

100. Short Answer: What is the limit of the fraction x/(x+1) as approaches infinity?

ANSWER: 1
Science Bowl Practice Questions Physics - 1

Science Bowl Practice Questions – Physics


1. Multiple Choice: For the hydrogen atom, which series describes electron transitions to the N=1 orbit,
the lowest energy electron orbit? Is it the:

w) Lyman series
x) Balmer series
y) Paschen series
z) Pfund series

ANSWER: W -- LYMAN SERIES

2. Multiple Choice: Electric current may be expressed in which one of the following units?

w) coulombs/volt
x) joules/coulomb
y) coulombs/second
z) ohms/second

ANSWER: Y -- COULOMBS/SECOND

3. Short Answer: In the SI system of measure, what is the unit of capacitance?

ANSWER: FARAD

4. Multiple Choice: A Newton is equal to which of the following?

w) kilogram-meter per second


x) meter per second squared
y) kilogram-meter per second squared
z) kilogram per meter-second

ANSWER: Y -- KILOGRAM-METER PER SECOND SQUARED

5. Multiple Choice: For an object moving in uniform circular motion, the direction of the instantaneous
acceleration vector is:

w) tangent to the path of motion


x) equal to zero
y) directed radially outward
z) directed radially inward

ANSWER: Z -- DIRECTED RADIALLY INWARD

6. Short Answer: A boy is standing on an elevator which is traveling downward with a constant velocity
of 30 meters per second. The boy throws a ball vertically upward with a velocity of 10 meters per
second relative to the elevator. What is the velocity of the ball, MAGNITUDE AND DIRECTION,
relative to the elevator shaft the instant the boy releases the ball?

ANSWER: 20 METERS PER SECOND DOWN

7. Multiple Choice: Work is equal to which of the following?

w) the cross product of force and displacement.


x) the product of force times time
y) force divided by time
z) the dot product of force and displacement

ANSWER: Z -- THE DOT PRODUCT OF FORCE AND DISPLACEMENT


Science Bowl Practice Questions Physics - 2

8. Multiple Choice: The work done by a friction force is:

w) always positive
x) always negative
y) always zero
z) either positive or negative depending upon the situation.

ANSWER: X -- ALWAYS NEGATIVE

9. Multiple Choice: As defined in physics, work is:

w) a scalar quantity
x) always a positive quantity
y) a vector quantity
z) always zero

ANSWER: W -- A SCALAR QUANTITY

10. Multiple Choice: A pendulum which is suspended from the ceiling of a railroad car is observed to hang
at an angle of 10 degrees to the right of vertical. Which of the following answers could explain this
phenomena?

w) The railroad car is at rest.


x) The railroad car is accelerating to the left.
y) The railroad car is moving with constant velocity to the right.
z) The railroad car is accelerating to the right.

ANSWER: X -- THE RAILROAD CAR IS ACCELERATING TO THE LEFT.

11. Multiple Choice: Two forces have magnitudes of 11 newtons and 5 newtons. The magnitude of their
sum could NOT be equal to which of the following values?

w) 16 newtons
x) 5 newtons
y) 9 newtons
z) 7 newtons

ANSWER: X -- 5 NEWTONS

12. Short Answer: A ball leaves a girl's hand with an upward velocity of 6 meters per second. What is the
maximum height of the ball above the girl's hand?

ANSWER: 1.8 METERS

13. Short Answer: A boy throws a ball vertically upward with a velocity of 6 meters per second. How long
does it take the ball to return to the boy's hand?

ANSWER: 1.22 SECONDS (accept: 1.2 seconds)

14. Short Answer: A toy train moves in a circle of 8 meters radius with a speed of 4 meters per second.
What is the magnitude of the acceleration of the train?

ANSWER: 2 METERS PER SECOND SQUARED

15. Short Answer: A certain machine exerts a force of 200 newtons on a box whose mass is 30 kilograms.
The machine moves the box a distance of 20 meters along a horizontal floor. What amount of work
does the machine do on the box?

ANSWER: 4000 JOULES


Science Bowl Practice Questions Physics - 3

16. Short Answer: A box is initially at rest on a horizontal, frictionless table. If a force of 10 Newtons acts
on the box for 3 seconds, what is the momentum of the box at the end of the 3 second interval?

ANSWER: 30 NEWTON-SECONDS or 30 KILOGRAM-METER PER SECOND

17. Multiple Choice: A block of metal which weighs 60 newtons in air and 40 newtons under water has a
density, in kilograms per meter cubed, of:

w) 1000
x) 3000
y) 5000
z) 7000

ANSWER: X -- 3000

18. Short Answer: A 10 kilogram body initially moving with a velocity of 10 meters per second makes a
head-on collision with a 15 kilogram body initially at rest. The two objects stick together. What is the
velocity of the combined system just after the collision?

ANSWER: 4 METERS PER SECOND

19. Short Answer: A certain spring is known to obey Hooke's Law. If a force of 10 newtons stretches the
spring 2 meters, how far will a 30 newton force stretch the spring?

ANSWER: 6 METERS
Science Bowl Practice Questions Physics - 4

20. Short Answer: A helicopter is ascending vertically with a constant speed of 6 meters per second
relative to the ground. At the instant the helicopter is 60 meters above the ground it releases a package.
What is the magnitude and direction of the velocity of the package, relative to the ground, the instant
the package is released by the helicopter?

ANSWER: 6 METERS/SECOND UP

21. Multiple Choice: If the distance between two objects, each of mass 'M', is tripled, the force of attraction
between the two objects is:

w) 1/2 the original force


x) 1/3 the original force
y) 1/9 the original force
z) unchanged

ANSWER: Y -- 1/9 THE ORIGINAL FORCE

22. Short Answer: A 40 kilogram girl climbs a vertical distance of 5 meters in twenty seconds at a constant
velocity. How much work has the girl done?

ANSWER: 2000 JOULES or 1960 JOULES

23. Short Answer: A machine performs 8 Joules of work in 2 seconds. How much power is delivered by
this machine?

ANSWER: 4 WATTS

24. Multiple Choice: In physics, a radian per second is a unit of:

w) angular displacement
x) angular velocity
y) angular acceleration
z) angular momentum.

ANSWER: X -- ANGULAR VELOCITY

25. Multiple Choice: If the resultant force acting on a body of constant mass is zero, the body's momentum
is:

w) increasing
x) decreasing
y) always zero
z) constant

ANSWER: Z – CONSTANT

26. Short Answer: What is the name of the first American physicist to win two Nobel prizes?

ANSWER: (JOHN) BARDEEN

27. Multiple Choice: Which of the following scientists is responsible for the exclusion principle which
states that two objects may NOT occupy the same space at the same time? Was it:

w) Heisenberg
x) Bohr
y) Teller
z) Pauli

ANSWER: Z -- PAULI
Science Bowl Practice Questions Physics - 5

28. Short Answer: Who shared the Nobel Prize in Physics in 1909 with Guglielmo Marconi for his
contribution to the development of wireless telegraphy?

ANSWER: (CARL FERDINAND) BRAUN

29. Short Answer: Who first theoretically predicted the existence of the positron, a positively charged
electron? He received the Nobel Prize in Physics in 1933.

ANSWER: (PAUL ADRIEN MAURICE) DIRAC

30. Short Answer: Name the female physicist who received the Nobel Prize in 1963 for her discovery
concerning the shell structure of the nucleus.

ANSWER: (MARIA GOEPPERT) MAYER

31. Short Answer: The constant potential difference across a 2 ohm resistor is 20 volts. How many watts of
power are dissipated by this resistor?

ANSWER: 200 WATTS

32. Short Answer: The potential difference across a 4 ohm resistor is 20 volts. Assuming that all of the
energy dissipated by this resistor is in the form of heat, how many joules of heat are radiated in 10
seconds?

ANSWER: 1000 JOULES

33. Multiple Choice: The force acting between two point charges can be computed using which of the
following laws?

w) Ohm's Law
x) Ampere's Law
y) Coulomb's Law
z) Newton's Second Law.

ANSWER: Y -- COULOMB'S LAW


Science Bowl Practice Questions Physics - 6

34. Short Answer: Five volts are applied across the plates of a parallel plate capacitor. The distance of
separation of the plates is .02 meters. What is the magnitude of the electric field inside the capacitor?

ANSWER: 250 VOLTS PER METER or 250 NEWTONS PER COULOMB

35. Short Answer: Used normally, a 150-watt, 120 volt light bulb requires how many amps of current?

ANSWER: 1.25 AMPS

36. Short Answer: If 10 joules of energy are required to move 5 coulombs of charge between two points,
the potential difference between the two points is equal to how many volts?

ANSWER: 2 VOLTS

37. Multiple Choice: Induced electric currents can be explained using which of the following laws?

w) Gauss's Law
x) Faraday's Law
y) Ohm's Law
z) Ampere's Law

ANSWER: X -- FARADAY'S LAW

38. Multiple Choice: For a negative point charge, the electric field vectors:

w) circle the charge


x) point radially in toward the charge
y) point radially away from the charge
z) cross at infinity

ANSWER: X -- POINT RADIALLY IN TOWARD THE CHARGE

39. Multiple Choice: For an infinite sheet of positive charge, the electric field lines:

w) run parallel to the sheet of charge


x) are perpendicular to the sheet of charge and point in toward the sheet
y) are perpendicular to the sheet of charge and point away from the sheet
z) fall off as one over r squared

ANSWER: Y -- ARE PERPENDICULAR TO THE SHEET OF CHARGE AND POINT AWAY


Science Bowl Practice Questions Physics - 7

40. Multiple Choice: Five coulombs of charge are placed on a thin-walled conducting shell. Once the
charge has come to rest, the electric potential inside the hollow conducting shell is found to be:

w) zero
x) uniform inside the sphere and equal to the electric potential on the surface of the sphere
y) smaller than the electric potential outside the sphere
z) varying as one over r squared.

ANSWER: X -- UNIFORM INSIDE THE SPHERE AND EQUAL TO THE ELECTRIC OTENT

41. Short Answer: A two farad and a four farad capacitor are connected in series. What single capacitance
is "equivalent" to this combination?

ANSWER: 4/3 FARADS

42. Multiple Choice: Three capacitors with different capacitances are connected in series. Which of the
following statements is TRUE?

w) All three of the capacitors have the same potential difference between their plates.
x) The magnitude of the charge is the same on all of the capacitor plates.
y) The capacitance of the system depends on the voltage applied across the three
capacitors.

ANSWER: X -- THE MAGNITUDE OF THE CHARGE IS THE SAME ON ALL OF


THE CAPACITOR PLATES

43. Multiple Choice: For a parallel-plate capacitor with plate area "A" and plate separation "d", the
capacitance is proportional to which of the following?

w) A divided by d squared
x) A times d
y) A divided by d
z) d divided by A

ANSWER: Y -- A DIVIDED BY D

44. Multiple Choice: A constant potential difference is applied across the plates of a parallel-plate
capacitor. Neglecting any edge effects, the electric field inside the capacitor is:

w) constant
x) varying as one over r squared
y) decreasing as one moves from the positive to the negative plate z) zero

ANSWER: W -- CONSTANT

45. Short Answer: A 10 farad capacitor is used in a circuit. The voltage difference between the plates of
the capacitor is 20 volts. What is the magnitude of the charge on each of the capacitor's plates?

ANSWER: 200 COULOMBS

46. Short Answer: A circuit which employs a DIRECT CURRENT source has a branch which contains a
capacitor. After the circuit has reached a steady state, what is the magnitude of the current in the circuit
branch which contains the capacitor?

ANSWER: THE CURRENT IS ZERO

47. Short Answer: A charged particle is moving in a UNIFORM magnetic field. If the direction of motion
of the charged particle is parallel to the magnetic field, describe the shape of the charged particle's path.

ANSWER: STRAIGHT LINE


Science Bowl Practice Questions Physics - 8

48. Multiple Choice: An infinitely long wire carries a current of three amps. The magnetic field outside the
wire:

w) points radially away from the wire


x) points radially inward
y) circles the wire
z) is zero.

ANSWER: Y -- CIRCLES THE WIRE

49. Multiple Choice: A copper rod which is 1 centimeter in diameter carries a current of 5 amps. The
current is distributed uniformly throughout the rod. The magnetic field half way between the axis of the
rod and its outside edge is:

w) zero.
x) pointing radially outward
y) pointing radially inward
z) circles the axis of the rod

ANSWER: Z -- CIRCLES THE AXIS OF THE ROD

50. Multiple Choice: Iron is what type of magnetic material? Is it:

w) diamagnetic
x) paramagnetic
y) ferromagnetic
z) non-magnetic

ANSWER: Y – FERROMAGNETIC
Science Bowl Practice Questions Physics - 9

51. Short Answer: The focal length of a concave mirror is 2 meters. An object is positioned 8 meters in
front of the mirror. Where is the image of this object formed?

ANSWER: 8/3 METER or 2.66 METERS IN FRONT OF THE MIRROR

52. Short Answer: A converging thin lens has a focal length of 27 centimeters. An object is placed 9
centimeters from the lens. Where is the image of this object formed?

ANSWER: -13.5 CENTIMETERS or 13.5 CENTIMETERS ON THE OBJECT SIDE


OF THE LENS

53. Short Answer: In Bohr's theory of the atom, what force was responsible for holding the electrons in
their orbit?

ANSWER: COULOMB FORCE or THE FORCE OF ATTRACTION BETWEEN THE PROTON


(NUCLEUS) AND THE ELECTRON

54. Short Answer: Davisson and Germer scattered electrons from a crystal of nickel. The scattered
electrons formed a strong diffraction pattern. What important conclusion was drawn from this
experiment?

ANSWER: ELECTRONS ACTED LIKE WAVES

55. Short Answer: The speed at which a wave propagates down a string is 300 meters per second. If the
frequency of this wave is 150 Hertz, what is the wavelength of this wave?

ANSWER: 2 METERS

56. Multiple Choice: A standing wave is formed on a tightly stretched string. The distance between a node
and an antinode is:

w) 1/8 wavelength
x) 1/4 wavelength
y) 1/2 wavelength
z) 1 wavelength

ANSWER: X -- 1/4 WAVELENGTH

57. Multiple Choice: When a physical property such as charge exists in discrete "packets" rather than in
continuous amounts, the property is said to be:

w) discontinuous
x) abrupt
y) quantized
z) noncontinuous

ANSWER: Y – QUANTIZED

58. Short Answer: Assume a ray of light is incident on a smooth reflecting surface at an angle of incidence
of 15 degrees to the normal. What is the angle between the incident ray and the reflected ray?

ANSWER: 30 DEGREES

59. Short Answer: The focal length of a concave spherical mirror is equal to 1 meter. What is the radius of
curvature of this mirror?

ANSWER: 2 METERS

60. Short Answer: A virtual image can be formed by one or more of the following single mirrors? Identify
them.
Science Bowl Practice Questions Physics - 10

w) plane mirror
x) concave spherical mirror
y) convex spherical mirror
z) all of the above

ANSWER: Z -- ALL OF THE ABOVE (accept: A, B and C)

61. Short Answer: A quarter of a wavelength is equal to how many degrees of phase?

ANSWER: 90 DEGREES

62. Multiple Choice: An organ pipe which is open at both ends resonates at its fundamental frequency.
Neglecting any end effects, what wavelength is formed by this pipe in this mode of vibration if the pipe
is 2 meters long?

w) 2 meters
x) 4 meters
y) 6 meters
z) 8 meters.

ANSWER: X -- 4 METERS

63. Multiple Choice: Whose principle or law states that each point on a wavefront may be considered a
new wave source? Is it:

w) Snell's Law
x) Huygen's Principle
y) Young's Law
z) Hertz's Law.

ANSWER: X -- HUYGEN'S PRINCIPLE

64. Short Answer: The frequency of a wave is 50 Hertz and its wavelength is 25 meters. What is the
velocity of this wave?

ANSWER: 1250 METERS/SECOND

65. Multiple Choice: The wave nature of light is demonstrated by which of the following?

w) the photoelectric effect


x) color
y) the speed of light
z) diffraction

ANSWER: Z -- DIFFRACTION

66. Multiple Choice: The collision between a photon and a free electron was first explained by which of
the following scientists?

w) Einstein
x) Heisenberg
y) Compton
z) Bohr

ANSWER: Y – COMPTON

67. Short Answer: Besides solid, liquid, and gas, what is the fourth form of matter?

ANSWER: PLASMA
Science Bowl Practice Questions Physics - 11

68. Short Answer: What is 25,000 miles per hour on earth, and 5,300 miles per hour on the Moon?

ANSWER: ESCAPE VELOCITY

69. Short Answer: In Einstein's universe, what is the fourth dimension?

ANSWER: TIME

70. Multiple Choice: The Tesla and the Gauss are units of measure of:

w) conductance
x) magnetic field strength
y) magnetic flux
z) electrical current

ANSWER: X -- MAGNETIC FIELD STRENGTH

71. Short Answer: Shockley, Brattain and Bardeen won a Nobel prize for what small invention?

ANSWER: TRANSISTOR

72. Short Answer: What mechanical and electronic device has a name derived from a Czechoslovakian
word meaning "work; compulsory service"?

ANSWER: ROBOT

73. Short Answer: What is the name of the temperature and pressure conditions at which water can be in
the solid, liquid and gas phases simultaneously?

ANSWER: TRIPLE POINT

74. Multiple Choice: Which of the following colors of visible light has the longest wavelength? Is it:

w) violet
x) green
y) yellow
z) red

ANSWER: Z – RED

75. Multiple Choice: A 10 kilogram mass rests on a horizontal frictionless surface. A horizontal force of 5
Newtons is applied to the mass. After the force has been applied for 1 second, the velocity of the mass
is:

w) 0 meters per second


x) 0.5 meters per second
y) 5 meters per second
z) 50 meters per second

ANSWER: X -- 0.5 METERS PER SECOND

76. Multiple Choice: A worker lifts a 10 kilogram block a vertical height of 2 meters. The work he does on
the block is:

w) 5 Joules
x) 20 Joules
y) 49 Joules
z) 200 Joules

ANSWER: Z -- 200 JOULES


Science Bowl Practice Questions Physics - 12

77. Multiple Choice: An impulse of 10 kilogram-meter per second acting on an object whose mass is 5
kilogram will cause a change in the objects velocity of:

w) 0.5 meters per second


x) 2 meters per second
y) 10 meters per second
z) 50 meters per second

ANSWER: X -- 2 METERS PER SECOND


Science Bowl Practice Questions Physics - 13

78. Multiple Choice: The time needed for a net force of 10 newtons to change the velocity of a 5 kilograms
mass by 3 meters/second is:

w) 1.5 seconds
x) 6 seconds
y) 16.7 seconds
z) 150 seconds

ANSWER: W -- 1.5 SECONDS

79. Multiple Choice: The value of G, the universal gravitational constant, was measured experimentally
by:

w) Newton
x) Cavendish
y) Copernicus
z) Kepler

ANSWER: X -- CAVENDISH

80. Multiple Choice: Two steel balls are at a distance S from one another. As the mass of ONE of the balls
is doubled, the gravitational force of attraction between them is:

w) quartered
x) halved
y) doubled
z) quadrupled

ANSWER: Y -- DOUBLED

81. Multiple Choice: If the distance between the earth and moon were halved, the force of the attraction
between them would be:

w) one fourth as great


x) one half as great
y) twice as great
z) four times as great

ANSWER: Z -- FOUR TIMES AS GREAT

82. Multiple Choice: As a 10 kilogram mass on the end of a spring passes through its equilibrium position,
the kinetic energy of the mass is 20 joules. The speed of the mass is:

w) 2.0 meters per second


x) 4.0 meters per second
y) 5.0 meters per second
z) 6.3 meters per second

ANSWER: W -- 2.0 METERS PER SECOND

83. Multiple Choice: As a longitudinal wave moves through a medium, the particles of the medium:

w) vibrate in a path parallel to the path of the wave


x) vibrate in a path perpendicular to the path of the wave
y) follow the wave along its entire path
z) do not move

ANSWER: W -- VIBRATE IN A PATH PARALLEL TO THE PATH OF THE WAVE

84. Multiple Choice: As a pendulum is raised to higher altitudes, its period:


Science Bowl Practice Questions Physics - 14

w) increases
x) decreases
y) remains the same
z) decreases, then remains the same

ANSWER: W -- INCREASES

85. Multiple Choice: Two vibrating particles that are "out of phase" differ in the phase of their vibration
by:

w) 1/4 cycle
x) 1/2 cycle
y) 3/4 cycle
z) 1 cycle

ANSWER: X -- 1/2 CYCLE

86. Multiple Choice: The SI unit of pressure is the:

w) Torr
x) dyne per centimeter squared
y) atmosphere
z) pascal

ANSWER: Z – PASCAL

87. Multiple Choice: An electroscope charged WITHOUT contacting a charged body is charged by:

w) induction
x) conduction
y) convection
z) insulation

ANSWER: W -- INDUCTION

88. Multiple Choice: The potential drop between the terminals of a battery is equal to the battery's EMF
when:

w) no current is drawn from the battery


x) a very large current is drawn from the battery
y) the internal resistance of the battery is very large
z) the resistance in the external circuit is small

ANSWER: W -- NO CURRENT IS DRAWN FROM THE BATTERY

89. Multiple Choice: To convert a galvanometer to a voltmeter, you should add a:

w) high resistance in series


x) high resistance in parallel
y) low resistance in series
z) low resistance in parallel

ANSWER: W -- HIGH RESISTANCE IN SERIES

90. Multiple Choice: The greatest induced EMF will occur in a straight wire moving at constant speed
through a uniform magnetic field when the angle between the direction of the wire's motion and the
direction of the magnetic field is

w) 0 degrees
x) 30 degrees
Science Bowl Practice Questions Physics - 15

y) 60 degrees
z) 90 degrees

ANSWER: Z -- 90 DEGREES

91. Multiple Choice: A 10 volt battery connected to a capacitor delivers a charge of 0.5 coulombs. The
capacitance of the capacitor is:

w) 2 x 10-2 farads
x) 5 x 10-2 farads
y) 2 farads
z) 5 farads

ANSWER: X -- 5 x 10-2 FARADS

92. Multiple Choice: Two light rays will interfere constructively with maximum amplitude if the path
difference between them is:

w) one wavelength
x) one-half wavelength
y) one-quarter wavelength
z) one-eighth wavelength

ANSWER: W -- ONE WAVELENGTH

93. Multiple Choice: Light is normally incident on a thin soap film and is reflected. If the wavelength of
this light is "L" and the index of refraction of the soap film is "N", complete destructive interference
will occur for a film thickness of:

w) L / 8N
x) L / 4N
y) L / 2N
z) 3L / 4N

ANSWER: Y -- L / 2N

94. Multiple Choice: The Michelson interferometer was designed to study the nature of:

w) water waves
x) sound waves
y) an "ether"
z) sunlight

ANSWER: Y -- AN "ETHER"

95. Multiple Choice: The Millikan experiment showed that electric charge was:

w) negative
x) quantized
y) positive
z) unmeasurable

ANSWER: X -- QUANTIZED

96. Multiple Choice: When a metal becomes a superconductor, there is a tremendous decrease in its:

w) total volume
x) electrical resistance
y) length
z) density
Science Bowl Practice Questions Physics - 16

ANSWER: X -- ELECTRICAL RESISTANCE

97. Multiple Choice: An x-ray photon collides with a free electron, and the photon is scattered. During this
collision there is conservation of:

w) momentum but not energy


x) neither momentum nor energy
y) energy but not momentum
z) both momentum and energy

ANSWER: Z -- BOTH MOMENTUM AND ENERGY

98. Multiple Choice: In the sun, helium is produced from hydrogen by:

w) radioactive decay
x) disintegration
y) fission
z) fusion

ANSWER: Z – FUSION

99. Multiple Choice: The half-life of an isotope of an element is 5 days. The mass of a 10 gram sample of
this isotope remaining after 20 days is:

w) 0.312 grams
x) 0.625 grams
y) 1.25 grams
z) 2.50 grams

ANSWER: X -- 0.625 GRAMS

100. Multiple Choice: The idea that electrons revolved in orbits around the nucleus of an atom without
radiating energy away from the atom was postulated by:

w) Thompson
x) Bohr
y) Rutherford
z) Einstein

ANSWER: X -- BOHR

You might also like